Exam Questions by Dupi Flashcards

1
Q

You see a 24-year old woman who states that she cannot receive the flu vaccine because she experiences a severe allergic reaction to eggs (i.e. respiratory distress, angioedema) requiring the use of epinephrine. The most appropriate response is:

a) Recommending vaccination with live attenuated influenza vaccine (LAIV)
b) Recommending vaccination with recombinant influenza vaccine (RIV)
c) Not recommending the flu vaccine until allergy testing is completed
d) Recommending the quadrivalent influenza vaccine every other year.

A

Ans: B

Recommending vac with recombinant influenza vaccine (RIV). RIV is egg-free and indicated for persons 18-49 years of age. All vaccines should be administered in settings in which personnel and equipment for rapid recognition and treatment of anaphylaxis are available. For those who report having a more severe reaction to eggs involving angioedema, resp distress, lightheadedness or emesis/ or those requiring Epinephrine, ACIP recommends RIV3. If RIV3 is not available, the patient should be referred to an allergist.

How well did you know this?
1
Not at all
2
3
4
5
Perfectly
2
Q

Secondary prevention is best defined as a focused effort to;

a) Prevent a health problem from occurring
b) Detect disease in an early, asymptomatic state to minimize its impact
c) Initiate treatment prior to a definitive diagnosis based on clinical experience and observations.
d) Optimize current treatment regimens to minimize negative disease-induced outcomes

A

Answer: B

Detect disease in an early, asymptomatic state to minimize its impact

How well did you know this?
1
Not at all
2
3
4
5
Perfectly
3
Q

You see a 76-year old woman living at home who is accompanied by her home care provider. She has COPD and T2DM. An example of a secondary prevention strategy is:

a) Administering the seasonal influenza vaccine
b) Checking her blood glucose level
c) Screening for physical or financial abuse
d) Adjusting her insulin dosing regimen

A

Answer: C Screening for physical or financial abuse is secondary prevention.

How well did you know this?
1
Not at all
2
3
4
5
Perfectly
4
Q

Three suicides have occurred at a university leading up to finals week. The student health center is offering on-on-one and small group counseling to any student who is feeling stressed. Meeting with students who are anxious and showing signs of stress is an example of:

a) Primary prevention
b) Secondary prevention
c) Community action
d) Tertiary prevention

A

Ans: D

Tertiary prevention involves efforts directed toward individuals in order to reduce the impact of dysfunction, disease, or distress and to reduce the duration of the condition. If students are already showing signs of distress, tertiary prevention minimizes the effect of that stress

How well did you know this?
1
Not at all
2
3
4
5
Perfectly
5
Q

According to the recommendation by the USPSTF, a 52-yr old woman should undergo a mammogram :

a) Every year
b) Every 2 years
c) Annually beginning at age 55
d) Every 5 years

A

Ans: B Every 2 years

How well did you know this?
1
Not at all
2
3
4
5
Perfectly
6
Q

Among men with no symptoms suspicious for prostate cancer, the USPTF recommends:

a) Annual PSA-based screening beginning at age 40
b) Biennial PSA-based screening beginning at age 45
c) Annual PSA-based screening beginning at age 50
d) Against PSA-based screening for prostate cancer

A

Ans: D

A grade-D recommendation for PSA-screening suggests that there is mod to high certainty that this service has no net benefit or that the harm outweighs the benefit. This rec is only for men who do NOT have symptoms suspicious of prostate cancer.

How well did you know this?
1
Not at all
2
3
4
5
Perfectly
7
Q

According to the USPTF, screening adults for depression should occur:

a) On an annual basis
b) Immediately following significant life events (unemployment, illness, etc.)
c) When staff-assisted depression care supports are in place to assure accurate diagnosis, effective treatment, and follow-up
d) Only for patients with multiple risk factors for depression

A

Ans: C

Routine screening should not occur when staff assisted depression care supports are not in place.

How well did you know this?
1
Not at all
2
3
4
5
Perfectly
8
Q

Which of the following statements is true regarding screening adolescent and adult males for testicular cancer?

a) Screening is not recommended
b) Screening by self-exam offers significant health benefits
c) Screening by clinician examination offers significant health benefits
d) Screening for testicular cancer should be conducted every 5 years starting at age 18 years.

A

Ans: A

Screening is unlikely to offer meaningful health benefits given the very low incidence and high cure rate of even advanced testicular cancer. The risk of false positives & anxiety is high.

How well did you know this?
1
Not at all
2
3
4
5
Perfectly
9
Q

Which of the following is least likely to be noted in a person with psoriasis vulgaris?

a) The face is typically involved
b) Lesions are often seen as well-demarcated plaques on the knees
c) Aggravating factors include stress, alcohol abuse, and cigarette smoking
d) Most milder cases can be successfully managed with a topical corticosteroid or vitamin D derivative

A

Ans: A

Psoriasis vulgaris rarely involves the face

How well did you know this?
1
Not at all
2
3
4
5
Perfectly
10
Q

A punch biopsy is best presented to the pathologist when it is obtained from:

a) The center of the lesion
b) The darkest spot of the lesion
c) The border between the lesion and unaffected skin
d) Any area of the lesion in question

A

Ans: C

The pathologist is evaluation the difference between normal and abnormal cells. A biopsy that contains both cell types is the most valuable for exam.

How well did you know this?
1
Not at all
2
3
4
5
Perfectly
11
Q

A reasonable alternative diagnosis for a patient with acne vulgaris would be:

a) Rosacea
b) Tinea facialis
c) Hydradenitis suppurativa
d) Seborrheic dermatitis

A

Ans: A

Acne and rosacea are characterized by similar appearance and distribution

How well did you know this?
1
Not at all
2
3
4
5
Perfectly
12
Q

When counseling for melanoma risk assessment and reduction, the NP advises the patient:

a) To avoid the sun between 9am and 3pm
b) That family history is not a significant factor
c) That serious burns in middle age confer greatest risk
d) To use sunscreen with sun protection factor of 45 or higher

A

Ans: A

Avoid sun exposure between 9a and 3p

How well did you know this?
1
Not at all
2
3
4
5
Perfectly
13
Q

Therapies for the treatment of rosacea in an otherwise healthy 54-year old woman include topical administration of al of the following except:

a) Clindamycin cream
b) Azelaic acid gel
c) Corticosteroid cream
d) Tretinoin cream

A

Ans: C

The goal for the treatment of Rosacea is to control flares rather than to cure the condition. Topical corticosteroids should be avoided on the face as it can produce a rosacea-like syndrome or can worsen pre-existing rosacea

How well did you know this?
1
Not at all
2
3
4
5
Perfectly
14
Q

The presence of abnormally shaped red blood cells is known as:

a) Anisocytosis
b) Reticulocytopenia
c) Poikilocytosis
d) Hypochromic RBCs

A

Ans: C

Poikilocytosis is the presence of abnormally-shaped red blood cells.

Anisocytosis is an increase in the normal variation in RBC size

How well did you know this?
1
Not at all
2
3
4
5
Perfectly
15
Q

One week into treatment for iron deficiency anemia, you anticipate which of the following:

a) Increase in the hemoglobin level by 2%
b) Normal ferritin
c) 12% increase in hematocrit
d) Reticulocytosis

A

Ans: D

Treatment for IDA may last several months and retic production peaks at about 6 days. Hemoglobin levels will increase about 2g/dL and Hct will increase 6%. Ferritin levels will reach normal in approx 4-6 months

How well did you know this?
1
Not at all
2
3
4
5
Perfectly
16
Q

Concerning the evaluation of a red blood cell folate level, which of the following is true:

a) Potentially falsely lowered in a person with rapidly developing folate deficiency
b) Influenced by dietary intake over the 24-hour period prior to the test
c) Remains stable throughout an individual RBC lifespan
d) Evaluates Vitamin C deficiency in a chronically ill person

A

Ans: C

Remains stable: Folate is incorporated in erythrocyte during cell development and does not change over the life of the cell. It is not immediately influenced by diet.

How well did you know this?
1
Not at all
2
3
4
5
Perfectly
17
Q

The most common cause of Pernicious Anemia is:

a) Occult blood loss
b) Intestinal Malabsorption
c) Dietary deficiency of B12
d) Reduced intrinsic factor production

A

Ans: D

Pernicious anemia is caused by a deficiency in Vitamin B12 due to an inability to absorb B12 from food due to reduced production of intrinsic factor in the stomach which causes B12 not to be absorbed in the small intestine.

How well did you know this?
1
Not at all
2
3
4
5
Perfectly
18
Q

Which of the following is the most consistent with Anemia of chronic disease:

a) Macrocytic RBCs
b) Elevated MCHC
c) Hematocrit > 24%
d) Decreased level of Ferritin

A

Ans: C

ACD is characterized by a reduced erythropoietin response that results in RBC hypo-proliferation. These patients have a hematocrit > 24%, slightly increased or normal Ferritin, normo-slightly microcytic RBCs and normal MCHC

How well did you know this?
1
Not at all
2
3
4
5
Perfectly
19
Q

In the person with reactive thrombocytosis, clotting risk is typically absent until a platelet count of:

a) > 400,000
b) >800,000
c) >1,000,000
d) >2,000,000

A

Ans: C
Normal is 150,000-450,000 (avg 250,000)
Reactive thrombocytosis is usually caused by acute bleeding, allergic reactions, cancer, infections, spleen removal, and some anemias. An increased risk of blood clots can occur once the platelet count is over 1 million

How well did you know this?
1
Not at all
2
3
4
5
Perfectly
20
Q

Criteria for “incident to” services include all of the following except:

a) An integral part of the patient’s treatment course
b) Commonly without charge or included as part of physician services
c) Of a type commonly furnished in an office or clinic rather than an institutional setting
d) Normally rendered for an additional yet minimal fixed fee

A

Ans: D

“Incident to” qualifications are:

1) the service is an integral part of the patient care,
2) it is commonly rendered without charge (height weight etc) and is part of physician billing
3) the service is commonly furnished in an office or a clinic (not an institution)
4) the physician must provide initial service and subsequent services at a rate that suggest active participation.

How well did you know this?
1
Not at all
2
3
4
5
Perfectly
21
Q

All of the following are required Medicare terms and conditions for paying NP services except:

a) The services are within the NP scope of practice
b) The services performed are those for which a physician would be able to bill medicare
c) The services are performed in collaboration with a physician
d) Separate charges are billed for NP services and facility charges

A

Ans: D

Separate charges are billed for NP services and facility charges

How well did you know this?
1
Not at all
2
3
4
5
Perfectly
22
Q

You see a 54-year-old man with chronic bronchitis who has smoked for the past 10 years. During this visit, he states “I set a quit date after my son’s wedding in 2 weeks”. According to Prochaska, this patient is at what stage of change:

a) Precontemplation
b) Contemplation
c) Preparation
d) Action

A

Ans: C

In preparation stage, the person has made a decision to change and plans to change within 30 days.

How well did you know this?
1
Not at all
2
3
4
5
Perfectly
23
Q

Which of the following is not a criterion for a level 4 office visit:

a) At least 4 elements of HPI; + or – responses to at least 2 ROS questions, and at least 1 notation in past history, family or social history.
b) At least 12 elements of physical examination
c) Medical decision-making of moderate complexity
d) Patient must be seen by a physician at some point during the visit.

A

Ans: D

There is no requirement for the patient to be seen by a physician during a level four office visit. All of the rest are true and at least 2 of the three must be met.

How well did you know this?
1
Not at all
2
3
4
5
Perfectly
24
Q

In severe eczema, patients tend to prefer _________ but providers should prescribe __________

a) Ointment/lotion
b) Cream/gel
c) Lotion/ointment
d) Gel/cream

A

Ans: C

Patients tend to prefer lotion because this form is soothing, easy to apply, and soaks in readily. However, severe eczema requires the highest potency with respect to both delivery and absorption; this is the ointment which delivers the greatest concentration of drug to the affected area.

How well did you know this?
1
Not at all
2
3
4
5
Perfectly
25
Q

A 59-year-old man complains of an occasional tremor, and he is concerned that he has Parkinson’s disease.
While there are no specific laboratory/diagnostic tests to confirm this patient’s suspicions, which of the
following would support the diagnosis?

A A habit of consuming nitrate-containing foods
B Kernig’ sign
C A history of alcohol abuse
D Myerson’s sign

A

Ans: D

Myerson’s sign is a tic-response to tapping above the nose and between the eyebrows. It is suggestive of Parkinson’s.

How well did you know this?
1
Not at all
2
3
4
5
Perfectly
26
Q

A 12-year-old boy presents to the clinic with several smalI, circular, erythematous lesions on his arm. The
lesions appear raised but have a central area of clearing. His mother reports that his sister has several similar ‘spots’ on her abdomen. Which of the following is the most likely diagnosis?

A Tinea versicolor
B Lyme disease
C Tinea corporis
D Molluscum contagiosa

A

Ans: C

most easily identifiable are the enlarging raised red rings with a central area of clearing (ringworm).

Treatment is topical zole

How well did you know this?
1
Not at all
2
3
4
5
Perfectly
27
Q

You are reviewing the chart of a patient who is terminally ill with bone cancer. A section of the chart pertains to the confidence interval (CI) of nausea for the patient. Which of the following statements is true about CI?

a) The smaller the confidence interval, the more precise the range of values.
b) The larger the confidence interval, the more precise the range of values
c) The smaller the confidence interval, the less the values deviate from the mean
d) The smaller the confidence interval, the more values deviate from the mean.

A

Ans: A

A confidence interval (CI) is a range or interval of values, between which an estimation of the unknown population parameter can be reliably estimated. The only true statement is that a small confidence interval implies a very precise range of values. Value deviation does not apply to a CI

How well did you know this?
1
Not at all
2
3
4
5
Perfectly
28
Q

A 7-year old male presents to clinic with a limp from pain on his right side. You ask how long he has been limping but the boy’s mother is not sure. She says that she noticed him limping a few days ago, but the boy did not complain of pain until very recently. He is unable to walk long distances. His temp is 99F, which the mom explains is due to a recent cold. Upon physical exam, you see no obvious signs, but an internal rotation of the hip causes a spasm. The radiography appears normal. Which of the following conditions do you suspect is the most likely cause?

a) Legg-Calve-Perthes disease
b) Toxic synovitis
c) Septic arthritis
d) Slipped capital femoral epiphysis

A

Ans: B

Hip spasm on internal rotation and recent viral infection indicate toxic synovitis (TS). The insidious onset of the signs and symptoms make septic arthritis unlikely as this condition has an acute onset.

Legg-Calves-Perthes disease is afebrile and less acute. The condition is apparent in radiograph studies and the physical finding include limited passive int rotation and abduction of the hip joint.

A patient with slipped capital femoris epiphysis will be unable to flex his hip as the femur abducts and will be afebrile. This also is acute onset

How well did you know this?
1
Not at all
2
3
4
5
Perfectly
29
Q

Your 57-year old male patient has awakened nightly for the last three weeks to severe throbbing pain around his left eye, particularly after drinking alcohol. The pain usually subsides within an hour. A physical exam reveals nothing too unusual, other than eye redness and rhinorrhea. Of the following, which would be the best way to manage the patient’s condition:

a) OTC analgesics
b) Clopidogrel
c) Ticlopidine
d) Sumatriptan

A

Ans: D

This patient has cluster headaches. A typical prescription for this condition is a triptan. Analgesics and/or relaxation is the treatment for tension headaches rather than the more severe cluster HA. Ticlopidine and Clopidogrel are prescribed for prophylaxis of TIA which has different presentation. Ticlopidine and clopidogrel are antiplatelet meds

How well did you know this?
1
Not at all
2
3
4
5
Perfectly
30
Q

A 65-year old fashion designer has in recent months, noticed persistent, sharp, and painful spasms in her left cheek. These incidents occur primarily when she is applying makeup and do not occur on the other side of her face. She describes the pain as feeling like an “electric shock”. Which of the following would be the least appropriate treatment for this condition:

a) Anti-seizure drugs
b) Tricyclic antidepressants
c) Calcium-channel blockers
d) Muscle relaxants

A

Ans: C

The sharp, painful spasms occurring on one side of the face are classic presentation of trigeminal neuralgia and a calcium channel blocker (CCB) does not treat this condition. CCB are most often used to treat HTN. Anti-seizure meds, muscle relaxants, and TCAs are commonly prescribed to diminish or prevent future spasmodic episodes.

How well did you know this?
1
Not at all
2
3
4
5
Perfectly
31
Q

A 7-yr-old patient is brought to you complaining of difficulty swallowing and drooling. During the exam, you note that the epiglottis is inflamed. Which of the following pathogens is usually NOT associated with this type of inflammation:

a) Streptococci
b) H. influenzae
c) Pneumococci
d) M. cararrhalis

A

Ans: D

While three common pathogens can cause epiglottitis, M. cararrhalis is often associated with sinusitis and acute OM. Pneumococci, Strep, and H. flu are all pathogens to consider when determining the cause of epiglottitis.

How well did you know this?
1
Not at all
2
3
4
5
Perfectly
32
Q

Danny is a 13-year-old AA male who is 5’6” and has a BMI of 34. His temp and BP are WNL. He has grown 2.5” in the last 12 months. He normally spends several hours each day in his room playing video games but in an effort to lose wt, he walks for an hour every evening. Today he began to feel pain in his groin which now has spread to his left thigh and knee as well. He denies any recent trauma. In your PE, you note that he is unable to properly flex either hip when the femur abducts. Which of the following dx is most likely:

a) Septic arthritis
b) Osgood-Schlatter disease
c) Slipped capital femoral epiphysis
d) Legg-Calve-Perthes disease

A

Ans: C

Danny is an adolescent, AA obese male who leads a sedentary lifestyle: all of which are common risk factors associated with slipped capital femoral epiphysis (SCFE). These factors, along with the acute onset of a limp with knee and thigh pain, the progression of pain bilaterally, and the presence of hip flexion difficulty indicate SFCE. Pain with this condition usually starts in the groin and spreads to the leg.

  • -Legg-Calve-Perthes disease also presents with an insidious, non acute, onset of a limp. However, the pain usually starts in the leg/calf and spreads to the groin.
  • -Osgood-Schlatter disease is associated with a rapid growth spurt and dies not typically present with hip pain. Septic arthritis may have an acute onset but would also present with high fever and warmth/swelling in the joint.
How well did you know this?
1
Not at all
2
3
4
5
Perfectly
33
Q

A 34-year-old real estate agent presents with what he initially assumed was a simple canker sore. Several more sores have appeared, however. You determine that these sores are actually brown spots. The same type of spots also appear on other parts of the patient’s body such as the knuckles and nail beds. After testing to confirm a dx of Addison’s Disease, which of the following would you use to treat this patient?

a) Glucocorticoids and mineralocorticoids
b) Reduction in the use of corticosteroids
c) Valacyclovir
d) Combination therapy, or active antiretroviral therapy

A

Answer: A

Glucocorticoids and mineralocorticoids: Addison’s occurs when there is Cortisol/glucocorticoid levels are low and these elements need replacement.

How well did you know this?
1
Not at all
2
3
4
5
Perfectly
34
Q

Which of the following findings would you expect to see in a patient with Cholelithiasis caused by biliary obstruction?

a) Sudden intensifying pain in the upper right abdomen, fever, dark urine, pale colored stools
b) Upper abdominal pain, nausea, vomiting, and oily, smelly stools
c) Mild to moderate aching abdominal pain in the lower left quadrant, loose stools, nausea, and vomiting
d) Vague colicky umbilical pain that shifts to the right lower quadrant, pain with right thigh extension, and nausea with one or two episodes of vomiting

A

Ans: A

Sudden intensifying pain in the upper right abdomen, fever, dark urine, pale colored stools

How well did you know this?
1
Not at all
2
3
4
5
Perfectly
35
Q

After reviewing Alison’s EKG and echo results, it is clear that her heart is unable to contract sufficiently, resulting in decreased CV output. Which of the following types of heart failure does this best describe?

a) Systolic
b) Diastolic
c) Left
d) Right

A

Ans: A

Systolic (filling)
Diastolic is “rest”

How well did you know this?
1
Not at all
2
3
4
5
Perfectly
36
Q

What is the purpose of Case Management:

a) To reduce and contain risks, cut costs, and improve efficiency of a clinic or institution
b) To establish the qualification and mastery of skills of nurse practitioners
c) To mobilize, monitor and control resources used by patients during illness
d) To improve the quality of health care through continuous monitoring and evaluation

A

Ans: C

To mobilize, monitor, and control resources used by patients during illness. Case management is a comprehensive and systematic approach to providing quality health care that balances the quality and cost of resources used by the patient during the course of his illness.

Risk management relates to A
Certification relates to B
Continuous Quality Improvement is D

How well did you know this?
1
Not at all
2
3
4
5
Perfectly
37
Q

Chris, a 62-year-old postal worker enters the ED and says that he thinks he has been exposed to anthrax. He believes that the exposure took place the previous day though he felt nothing at the time. He went home and awoke this morning feeling feverish with muscle pain and breathlessness. Which of the following is the most appropriate response:

a) Isolate the patient and administer an anthrax vaccine in combination with a 60-day antibiotic treatment plan.
b) Isolate the patient immediately to keep him from spreading the illness to others
c) Order a Gram Stain and alert local law enforcement if needed
d) Immediately order antibiotics such as Penicillin or Ciprofloxacin

A

Ans: C

Do a gram stain and alert local law enforcement if needed.

FYI: Anthrax is a Gram + rod

How well did you know this?
1
Not at all
2
3
4
5
Perfectly
38
Q

A 30-year-old AA patient complains of being constantly out of breath in the absence of strenuous physical activity. He has also noticed the sudden onset of severe pain in his back and chest. Lab tests show a Hemoglobin level of 13g/dL. Given your suspicion of anemia, you know that which of the following would be LEAST appropriate to order as a course of treatment?

a) IV fluids
b) Analgesics
c) Oxygen
d) 5 units of red blood cells

A

Ans: D

5 units of RBCs

How well did you know this?
1
Not at all
2
3
4
5
Perfectly
39
Q

During a physical exam, Brett mentions that he wants to quit his job but is worried about losing his health insurance. You inform him that the HIPAA will provide protection for workers when they change or lose their jobs. This includes certain private information that pertains to Brett’s experience with the healthcare system. What is NOT true about HIPAA?

a) All sharing of a patient’s information requires written authorization
b) Patient info stored in a health insurer’s computer is always protected information
c) Health care clearing houses are covered under the bill
d) Conversations among healthcare providers about one’s care or treatment are protected information.

A

Ans: A

All sharing of a patient’s information requires written authorization

How well did you know this?
1
Not at all
2
3
4
5
Perfectly
40
Q

A 56-year-old marketing exec has various past blood pressure readings between 140-150 systolic and 80-100 diastolic over the last few months. During an office visit, you find his current BP is 142/88. This collective data confirms a diagnosis of HTN. Based on national guidelines and the patient’s current BP, how would you classify his HTN today

a) Normal
b) Stage I
c) Stage II
d) Pre-Hypertension

A

Ans: B Stage I

Stage one is either a sustained elevation of systolic blood pressure (SBP) 140 - 159 mm Hg or a DBP 90-99

Other stages of HTN are:
normal 160 and > 100

How well did you know this?
1
Not at all
2
3
4
5
Perfectly
41
Q

A man who recently attempted suicide is brought to your office by his wife. Having conducted a significant amount of research on her own about suicide prevention, she has many questions. Which of the following choices is the LEAST effective treatment for her husband?

a) Antipsychotics if the patient is psychotic
b) Antidepressants if the situation is chronic
c) Intervention if the patient’s risk is escalating
d) Hospitalization if the situation is acute.

A

Ans: C Intervention if the patient’s risk is escalating

How well did you know this?
1
Not at all
2
3
4
5
Perfectly
42
Q

A mother brings her 5-month old son for a check. She is concerned that his right leg appears shorter than his left. She states that his right leg looks as if it were turned outward. She adds that it does not seem as if her child is in any pain. Suspicious, the nurse examined the child by adducting his right hip while applying pressure on the knee. For which of the following conditions is the NP examining?

a) Genu valgum
b) Genu varum
c) Slipped capital femoral epiphysis
d) Hip dysplasia

A

Ans: D Hip dysplasia

This is the Barlow Maneuver

Ortolani is rotate “OUT” away from center; abduction
Barlow is rotate “BACK” to the center; adduction

How well did you know this?
1
Not at all
2
3
4
5
Perfectly
43
Q

When Alison was 49-years-old, she had a scare when she discovered a lump in her breast. It was determined to be benign but since then, she has been diligent about monitoring herself for other lumps. She has also made sure to receive mammograms annually. Now, at 65, she is asking when these exams will end. What should you tell her?

a) This year
b) Ten years from now
c) Five years from now
d) Fifteen years from now

A

Ans: B

ten years from now. Screening guidelines state that mammograms continue through 74 so they STOP at 75

How well did you know this?
1
Not at all
2
3
4
5
Perfectly
44
Q

A 23-year-old male presents with inflammation and itching of the penile head. He reports being in a long-term, monogamous relationship and states that he knows that he and his partner have been faithful for their entire relationship. Which of the following is the most likely diagnosis and treatment?

a) Tinea cruris; topical antifungal
b) Lichen planus: topical corticosteroid
c) Candida balanitis; topical antifungal
d) Tinea capitis; oral antifungal

A

Answer: C

Candida balanitis with topical antifungal

How well did you know this?
1
Not at all
2
3
4
5
Perfectly
45
Q

Which of the following GYN cancers has the highest mortality rate?

a) Ovarian
b) Cervical
c) Uterine
d) Vaginal

A

Ans: A Ovarian

Uterine is the most common form of gyn cancer with more than 43,000 annual incidences. Ovarian cancer while affecting half that number, cause more deaths than all other gyn cancers combined mostly because there is no screening for ovarian cancer. Cervical CA is the third most common while vaginal cancer is the rarest.

How well did you know this?
1
Not at all
2
3
4
5
Perfectly
46
Q

Although a chest X-ray is not necessary during an asthma attack, the NP would expect to see which of the following on the film if one were taken?

a) Low, elevated diaphragm
b) Blunting of the costophrenic angle
c) Moderate hyperinflation
d) Minor infiltrates

A

Answer: C

Moderate hyperinflation

How well did you know this?
1
Not at all
2
3
4
5
Perfectly
47
Q

A 25-year-old presents with a bubo in the right groin as well as stiffness and aching near his genitals. You note that there is swelling in his right inguinal area. You aspirate the bubo and prescribe Doxy. For what condition would this be the proper treatment?

a) HSV type 2
b) Lymphogranuloma venereum
c) Molluscum contagiosum
d) Genital warts

A

Ans: B

Lymphogranuloma venereum

How well did you know this?
1
Not at all
2
3
4
5
Perfectly
48
Q

A 25-year-old male visits clinic complaining of external burning and itching on his penis. He reports that his condom broke during recent sex and he has since switched to a different brand. There are rumors that the partner he was with has HPV. He is worried that he may have been infected with the virus. Which of the following options is the best initial course of action?

a) Test for GC
b) Assume nothing, just analyze a blood sample
c) Test for HPV
d) Skin test to check for latex and/or spermicidal allergy

A

And: D

Skin test to check for latex/spermicidal allergy

How well did you know this?
1
Not at all
2
3
4
5
Perfectly
49
Q

Peter comes to clinic after having stood for four straight days. He complains of intermittent numbness and tingling on his right foot up to his calf as well as pain in that calf. During exam, you have Peter walk on his heels and you note pain and weakness of the dorsiflexion mechanism of the great toe and foot. Which of the following is our most likely suspicion?

a) L3-L4 disk pathology
b) L5-S1 Disk pathology
c) T5-T6 pathology
d) L4-L5 disk pathology

A

Ans: D

L4-L5 disk pathology

How well did you know this?
1
Not at all
2
3
4
5
Perfectly
50
Q

A 7-year-old is brought in complaining of difficulty swallowing and drooling. During the exam, you note that the epiglottis is inflamed. Which of the following pathogens is usually not associated with this type of inflammation?

a) Streptococci
b) H. flu
c) Pneumococci
d) M. catarrhalis

A

Ans: D

M catarrhalis: this organism is usually associated with AOM and sinusitis…..URI not epiglotitis

How well did you know this?
1
Not at all
2
3
4
5
Perfectly
51
Q

Joey, 31, complains of a few painless, “smooth, raised bumps” on his penis. He says that the bumps are fairly firm and match the color of his skin but do not hurt at all. Without further exam of the patient, with what condition can you diagnose the patient?

a) Genital warts
b) HSV type 2
c) Molluscum contagiosum
d) Gonorrhea

A

Ans: C

Molluscum contagiosum. There are very few sexually transmitted infections known to cause painless lesions. MC is viral and otherwise called “water warts”. Treatment is imiquimod.
HSV is painful, Warts are painless lesions but are typically soft and papillary rather than smooth and raised. GC often presents with white/yellow-green discharge but is not associated with lesions.

How well did you know this?
1
Not at all
2
3
4
5
Perfectly
52
Q

While treating 29-year-old Sue for vulvovaginitis, you inquire about her sexual history. She states that she has been sexually active for about 10 years. There are many causes of vulvovaginitis, but only one is a sexually transmitted disease. Which of the following is it?

a) GC
b) Chancroid
c) Chlamydia
d) Trichomoniasis

A

Ans: D: Trichomoniasis

How well did you know this?
1
Not at all
2
3
4
5
Perfectly
53
Q

Melanie, an 8-month-old comes to clinic with bluish skin and a cleft palate. The NP notes that the baby has twitching around her mouth with spasms of her hands and arms. Melanie’s parents also report seizures. Upon further observation, the NP learns that the baby is also developmentally delayed in many areas. Which of the following should be a part of your differential?

a) Turner syndrome
b) Marfan syndrome
c) DiGeorge Syndrome
d) Tay-sachs disease

A

Ans: C

DiGeorge Syndrome

How well did you know this?
1
Not at all
2
3
4
5
Perfectly
54
Q

Alison, 77, reports abdominal pain in the lower left side of her abdomen. In the last few weeks, she has experienced frequent constipation and has taken laxatives for relief. However, her abd pain has become increasingly severe. Her labs show a SED rate of 45mm/hr and leukocytosis. Additionally, a sigmoidoscopy reveals inflamed mucosa. Based on her findings and labs, which of the following is the most likely diagnosis?

a) IBS
b) Bowel obstruction
c) Diverticulitis
d) Ulcerative colitis

A

Ans: C

Diverticulitis

How well did you know this?
1
Not at all
2
3
4
5
Perfectly
55
Q

A 27-year-old female comes to clinic and explains that she has recently felt an itching and inflammation in her vagina, despite being sexually inactive for several months. Upon exam, you note vulvovaginal erythema. In addition, you discover a thick white curd-like discharge. Which of the following is she most likely experiencing?

a) Candidiasis
b) Trichomonas
c) BV
d) HIV

A

Ans: A

Candidiasis

How well did you know this?
1
Not at all
2
3
4
5
Perfectly
56
Q

A 24-yr-old comes to clinic with a sore throat, nasal congestion, and a cough. The patient repeatedly clears her watery congestion into a tissue. The patient does not have a fever and the PE is unremarkable. Given only this information, what is the most likely cause of her condition?

a) Influenza
b) Common cold
c) Mononucleosis
d) Tonsillitis

A

Ans: B

The common cold aka rhinosinusitis

How well did you know this?
1
Not at all
2
3
4
5
Perfectly
57
Q

Michelle, 6, comes to clinic with a high fever. Immediately upon entering the exam room, she suddenly loses consciousness, has uncontrolled jerky motions, and urinary incontinence. How should the child’s symptoms be initially managed?

a) Lumbar puncture
b) Lorazepam and diazepam
c) Maintain airway and administer oxygen
d) CPR

A

Ans: C

Maintain airway and administer Oxygen

How well did you know this?
1
Not at all
2
3
4
5
Perfectly
58
Q

Jason, 35, is being treated for HIV and will be in the hospital for a few days. His medical bills are being paid by his Aunt who wants to know why her nephew is hospitalized but Jason is not willing to share that information. Of the following, which is the most appropriate action for the NP to take?

a) The NP should forward the aunt’s request to the hospital ethics committee
b) The NP should not release the details of Jason’s health status to his aunt
c) The NP should encourage Jason and his aunt to work out their differences before disclosing any patient details
d) Since Jason’s aunt is paying the bill, she has the right to his medical information and the NP should provide it.

A

Ans: B

The NP should not release the details of Jason’s health status to the aunt

How well did you know this?
1
Not at all
2
3
4
5
Perfectly
59
Q

A child’s serology results come back showing Anti-HCV and HCV RNA. Which of the following may be included in his treatment plan?

a) Oxycodone
b) Interferon and ribavirin
c) Erythromycin
d) Trimethoprim-Sulfamethoxazole

A

Ans: B

Interferon and ribavirin

How well did you know this?
1
Not at all
2
3
4
5
Perfectly
60
Q

There is concern at your clinic that some children with developmental delays are not being identified soon enough. You are examining a 6 year old male. Of the following, which is the best tool to identify risk for developmental delay in this child?

a) Cognitive adaptive test
b) Denver II
c) Ages and Stages Questionnaire
d) M-CHAT

A

Ans: B

Denver II through age 6 years.

How well did you know this?
1
Not at all
2
3
4
5
Perfectly
61
Q

You are working on a study comparing the differences between patients with glaucoma and those with cataracts. Which of the following tests would be the best to employ to test differences in this case?

a) T-test
b) F-test
c) Wald test
d) A-test

A

Ans: A

T test a T-test is used to determine if two sets of data are significantly different from each other.

An F-test is used in ANOVA: analysis of variance between data sets.

How well did you know this?
1
Not at all
2
3
4
5
Perfectly
62
Q

A 54-year-old former pro athlete is being screened for HTN which runs in his family. Which of the following findings is NOT consistent with a diagnosis of significant HTN?

a) Blurred vision
b) An S4 heart sound
c) A subparietal headache
d) Epistaxis

A

Ans: C

HTN presents with an occipital headache.

How well did you know this?
1
Not at all
2
3
4
5
Perfectly
63
Q

The term “pervasive developmental disorders” refers to a group of conditions that involve delays in the development of may basic skills. Children with these conditions often are confused in their thinking and generally have problems understanding the world. Which of the following is a factor leading to this condition?

a) An x-chromosome break
b) No known factor
c) Lack of maternal infant bonding
d) Failure of a parent to respond to their infant’s cues

A

Ans: B

No known factor

How well did you know this?
1
Not at all
2
3
4
5
Perfectly
64
Q

You have been treating Chris for Asthma. You first prescribed a Short acting B2 adrenergic agonist. After a month or so, he reported that there had been little improvement, so you added an ICS to the regime. Now, after another month, you note that Doug’s asthma does not present with secretions, but he states that his symptoms still have not markedly improved. Of the following, which should you prescribe next?

a) Antileukotrienes
b) Inhaled anticholinergics
c) Ipratropium bromide
d) Long-acting B2 adrenergic agonist

A

Ans: D

LABA

How well did you know this?
1
Not at all
2
3
4
5
Perfectly
65
Q

A 30-year-old presents with a cough and headache. She reports that she also has been wheezing for nearly a full week. Upon examination, you note a low grade fever and bilateral rhonchi; however, after coughing, her lungs are relatively clear to auscultation. Of the following, which is the most likely diagnosis?

a) Asthma
b) Pneumonia
c) Emphysema
d) Acute Bronchitis

A

Ans: D

Acute Bronchitis

How well did you know this?
1
Not at all
2
3
4
5
Perfectly
66
Q

An 18-month-old has decreased appetite and her mom thinks she may be having abdominal pain. She has also been unusually tired. After a thorough history, you find that the family has been renovating their 1950s home. The child’s hemoglobin is 11.2 and the lead level is 46 mcg/dL. What is the next step?

a) Reevaluate the child in 1 month
b) Referral to hematology
c) Order a blood transfusion
d) Evaluate other children in the home

A

Ans: B

Referral to hematology for a lead level > 45. Chelation is indicated

How well did you know this?
1
Not at all
2
3
4
5
Perfectly
67
Q

During an annual exam, you notice that a 12-year-old Lisa has breast enlargement without separate nipple contour. You tell her that she can expect which of the following in the next stage of her development?

a) Sparse pale fine pubic hair
b) Breast buds with areolar enlargement
c) Receding areolas and retracting nipples
d) Onset of menses

A

Ans: D

Onset of menses. Lisa is at Tanner III

How well did you know this?
1
Not at all
2
3
4
5
Perfectly
68
Q

A 75-year-old diabetic male requires treatment for pneumonia. Which of the following drugs is NOT recommended for this patient?

a) A respiratory fluoroquinolones
b) Moxifloxacin
c) Gemifloxacin
d) A Macrolide antibiotic

A

Ans: D

Macrolides are not given to elders due to Q-T prolongation.

How well did you know this?
1
Not at all
2
3
4
5
Perfectly
69
Q

David and Sally want to make their home as safe as possible for their toddler. They come to clinic asking for advice. When discussing safety, which of the following is the best advice to give?

a) It is crucial to nail into place all window guards
b) Test the temperature of hot water before giving a bath
c) Train the toddler to stay off the stairs
d) Talk to the toddler about staying away from poisons

A

Ans: B

Test the temperature of hot water before giving a bath.

How well did you know this?
1
Not at all
2
3
4
5
Perfectly
70
Q

Donna, 25, had a positive pregnancy test one week ago. Her mom is currently on insulin therapy to control DM and Donna has a BMI of 31. Since she is in her first trimester, which of the following diagnostic tests should you first order to assess the risk specific to this patient?

a) CBC
b) Urinalysis
c) Chorionic villus sampling
d) Hemoglobin and hematocrit

A

Ans: B

Urinalysis

How well did you know this?
1
Not at all
2
3
4
5
Perfectly
71
Q

Using the Dubowitz/Ballard exam to estimate gestational age of a caucasian newborn, you determine him to be post term. Which of the following is a finding NOT related to a criterion included in this exam?

a) Plantar creases over the sole of the feet
b) Elasticity of cartilage in the nose
c) Hypertonic flexion of knees
d) Thickness and size of breast tissue

A

Ans: B

Elasticity of cartilage in the nose

How well did you know this?
1
Not at all
2
3
4
5
Perfectly
72
Q

A newly pregnant 23 woman comes to clinic. She has been experiencing food cravings, including a desire to consume ice. You suspect a specific type of anemia. Which of the following findings would you most expect as a result?

a) Low hemoglobin / high serum ferritin
b) Low hemoglobin / normal ferritin
c) High hemoglobin / high total iron binding capacity
d) Low hemoglobin/high total iron binding capacity

A

Ans: D

Low hemoglobin and High total iron binding capacity

How well did you know this?
1
Not at all
2
3
4
5
Perfectly
73
Q

Samantha, 15, has been dealing with asthma since she was 4 years old. She is now showing signs of HTN. She asks your advice about management of the condition and expresses concerns about any prescription that may cause her to wheeze. With this concern in mind, which of the following should you NOT advise her to prescribe for her new condition?

a) Beta blocker
b) Calcium channel blocker
c) Angiotensin converting enzyme inhibitors
d) Angiotensin II receptor blocker

A

Ans: A

Beta Blocker = wheezing
ACE = cough

How well did you know this?
1
Not at all
2
3
4
5
Perfectly
74
Q

Elly, 21, seeks counseling on contraceptive methods. She plans on having children within the year but would like a convenient, long-acting contraceptive method. Which of the following contraceptive methods is relatively contraindicated for this patient

a) Oral contraceptives
b) Ortho Evra
c) Injected contraception
d) Contraceptive ring

A

Ans: C

Injected contraception

How well did you know this?
1
Not at all
2
3
4
5
Perfectly
75
Q

A mother has just given birth to an infant daughter. The mother’s chart indicated that she is positive for HBsAg. Regarding Hepatitis B vaccination, which of the following is the correct action to take?

a) Administer the Hep B vaccine and 0.5 ml Hep B immune globulin within 12 hours
b) Test for HBsAg and antibody to HBsAg in the infant
c) Administer the Hep B vaccine before the child discharges and follow up with another two vaccinations given at 2 and 6 months of age
d) Arrange an appointment for the mother to bring her daughter in for Hep B vaccine when the child is 1-2 months of age

A

Ans: A

Administer the Hep B vaccine and 0.5ml Hep B immune globulin within 12 hours

How well did you know this?
1
Not at all
2
3
4
5
Perfectly
76
Q

A 30-yr-old female presents to clinic with frequent and debilitating headaches that are particularly intense about her neck. She says that her neck and shoulders feel extremely tight and tense and during these headaches. However, she denies any neurologic deficits. Based upon her complaints, what type of headache is this patient most likely experiencing?

a) Vascular
b) Migraine
c) Cluster
d) Tension

A

Ans: D

Tension

How well did you know this?
1
Not at all
2
3
4
5
Perfectly
77
Q

Carrie, 74, comes to clinic for a check up. During the interview, she mentions that her eye sight has deteriorated drastically. She states, “although I am old and have less than 20/20 vision, my eye sight has become cloudy, and i sometimes see halos around lights”. Carrie denies pain when asked. Accounting for her age and stated symptoms, which of the following would best serve to confirm the most likely diagnosis?

a) Single-eye diplopia
b) Constriction of visual fields
c) Dilated pupils
d) Halos around lights

A

Ans: A

Single eye diplopia is characteristic of Cataracts

How well did you know this?
1
Not at all
2
3
4
5
Perfectly
78
Q

You are treating a 67-year-old Latina. Based on her profile and family history, you decide to order a bone density to check for Osteoporosis. Which of the following is NOT a typical risk factor that would lead you to suspect osteoporosis in this patient?

a) Hyperthyroidism
b) Obesity
c) Excessive Alcohol use
d) Smoking

A

Ans: B

Obesity is not a risk factor for osteoporosis.

How well did you know this?
1
Not at all
2
3
4
5
Perfectly
79
Q

A 59-year-old male is having a follow up evaluation 2 years after the successful completion of chemo for leukemia. He says that he has been feeling run down and reports unexplained weight loss, pain below the ribs on his left side as well as night sweats. You examine him and determine that he also has a fever. You consider Chronic Myelogenous leukemia. Which of the following results is considered the hallmark of this disease?

a) Non-circulating blast cells in the bone marrow
b) Lymphocytosis
c) Philadelphia chromosome in leukemic cells
d) Pancytopenia with circulating blasts.

A

Ans: C

Philadelphia chromosome is hallmark in the diagnosis of CML

How well did you know this?
1
Not at all
2
3
4
5
Perfectly
80
Q

A 66-year-old female has a history of varicose veins and presents to clinic complaining of aching and water retention in her legs during the last few months. She also complains of night cramps in her lower extremities. Which of the following disorders best characterizes her symptoms?

a) Superficial thrombosis
b) Sciatica
c) Peripheral Vascular disease
d) Chronic venous insufficiency

A

Ans: D

Chronic venous insufficiency

How well did you know this?
1
Not at all
2
3
4
5
Perfectly
81
Q

Kenny, 24, works for a package delivery service. During the course of his workday, he repetitively lifts heavy items. He has a history of muscle strain and comes to clinic with a particularly severe strain that has kept him out of work for the past 2 days. You note slight edema and ecchymosis in his right upper back as well. X-ray reveals no rib fractures or bone injuries. Of the following, all are advisable in managing this patient except:

a) Oxycodone
b) Prevention education
c) Immobilization and ice
d) NSAIDs

A

Ans: A

Oxycodone is not indicated

How well did you know this?
1
Not at all
2
3
4
5
Perfectly
82
Q

Kaitlin, 18, has recently become sexually active. She requests a contraceptive from your clinic. A PE indicated that she is 5’6” and 205 pounds. Based on her height and weight, which would be the least effective form of contraception?

a) Ortho Evra
b) NuvaRing
c) Oral contraceptives
d) Diaphragm

A

Ans: A

The patch is not indicated for those with a high BMI though i have no idea what the cut-off is. Seems like 205 is pretty big for a kid!

How well did you know this?
1
Not at all
2
3
4
5
Perfectly
83
Q

Autism in kids is generally identified by 3 years of age. Alice is 30 months and her mom mentions that she is not playing with others, is unable to pretend play, has a very limited vocabulary. Her only interest is in stacking blocks at which she is very good. When considering the disorders in the spectrum of pervasive development disorders, which of the following should be your initial consideration in your differential?

a) Autism
b) Asperger’s syndrome
c) Fragile X
d) Down Syndrome

A

Ans: A

Autism

Asperger kids are usually very verbal, bright, and energetic
Fragile X does not present until puberty
Downs? well whatever

How well did you know this?
1
Not at all
2
3
4
5
Perfectly
84
Q

Of the following, which is a necessary guideline for interviewing adolescents in a clinical setting?

a) Have the parents and the adolescent in your office for a meeting
b) Having the adolescent fill out a questionnaire to save time
c) Advising the adolescent that his or her parents are privy to what was discussed in the interview
d) Excluding the parents from some of the interview with the adolescent

A

Ans: D

Excluding the parents from some of the interview with the adolescent

How well did you know this?
1
Not at all
2
3
4
5
Perfectly
85
Q

A patient, 43, visits clinic complaining of severe suboccipital pulsating headaches occurring in the morning and fading throughout the day. He is somewhat overweight and smokes. Which of the following should you most likely suspect as the primary diagnosis?

a) Migraine headache
b) Hypertension
c) Heart failure
d) Brain Tumor

A

Ans: B

Hypertension

How well did you know this?
1
Not at all
2
3
4
5
Perfectly
86
Q

During a clinic visit, Alex, 27, reports an adverse reaction to Mesalamine suppositories and requests an alternative treatment. Which of the following conditions does Alex have?

a) Diverticulitis
b) IBS
c) PUD
d) Ulcerative Colitis

A

Ans: D

Ulcerative colitis

How well did you know this?
1
Not at all
2
3
4
5
Perfectly
87
Q

Joe, 21, comes to clinic complaining of a painful bump with a small red-looking halo on his penis. The affected area is about 5mm in diameter, soft and tender to the touch. He states that the halo has been there for about a week and is VERY painful. He doubts that he has a STI since the only unprotected sex he has ever had is with his L-T girlfriend. He adds that his girlfriend can be trusted and that she has never had an STI. Given this information, the most likely diagnosis is?

a) Chlamydia
b) Genital warts
c) Gonorrhea
d) Chancroid

A

Ans: D

Chanroid for Syphillis

How well did you know this?
1
Not at all
2
3
4
5
Perfectly
88
Q

Gary, 66 reports thin stools and recurring bouts of constipation or diarrhea. He has also experienced rapid and unexplained weight loss in the last month. Gary is concerned because he has a family history of cancer. Of the following, which procedure would you order to best rule out colon cancer?

a) Sigmoidoscopy
b) Barium studies
c) Colonoscopy
d) Complete Blood Count

A

Ans: C

Colonoscopy

How well did you know this?
1
Not at all
2
3
4
5
Perfectly
89
Q

An HR Rep is asking about your 55 year old patient, Alison, who has gone through extensive treatment for alcoholism. The HR person says that since the company she works for is paying for her treatment, it would like to know whether she has been successful in treatment. How do you best respond to the company rep?

a) You say that any information about the patient would need written approval from her
b) You say you will forward the request to your supervisor
c) You say that though you would be happy to share the good news, you cannot because Alison first need to sign a release form
d) You say that you would be happy to share the good news and that indeed, Alison is improving.

A

Ans: A

Information about Alison would need written approval from her

How well did you know this?
1
Not at all
2
3
4
5
Perfectly
90
Q

Steve, 48, has been brought to clinic for alcoholism treatment. Steve, however, denies being an alcoholic. At this point, which of the following questions is the most effective one to ask this patient?

a) Do you ever drink to feel more attractive or charismatic?
b) When did you start drinking
c) Have you ever felt the need to cut down on your drinking?
d) How much money have you spent on alcohol in the past

A

Ans: C

Have you ever felt the need to cut down on your drinking?

How well did you know this?
1
Not at all
2
3
4
5
Perfectly
91
Q

The MMR vaccine must be administered in 2 doses. When should the MMR vaccines typically be given?

a) First dose: 1-2 years old; second dose: 15 years
b) First dose: 1-4 years; second dose 10 years
c) First dose: 12-15 months; second dose 6 years
d) Both doses given between the ages of 13 and 18 years

A

Ans: C

First dose between 12-15 months with the second dose at 6 years

Varicella is the other vaccine that is not given until 12 months.

How well did you know this?
1
Not at all
2
3
4
5
Perfectly
92
Q

An HIV positive male comes to clinic complaining of SOB and a serious cough. Given this information, which condition should you be most cautious to monitor for in this patient?

a) Pneumocystis jiroveci pneumonia
b) Mycoplasma pneumoniae
c) Cytomegalovirus
d) Acute Bronchitis

A

Ans: A

Jiroveci pneumonia

How well did you know this?
1
Not at all
2
3
4
5
Perfectly
93
Q

Which of the following best describes a II/VI heart murmur?

a) Moderately loud; easily heard
b) Barely audible
c) Audible but faint
d) Loud; associated with a thrill

A

Ans: C

Audible but faint
III is louder and IV has a thrill

How well did you know this?
1
Not at all
2
3
4
5
Perfectly
94
Q

Alison, a 30 year-old female is worried about the “mood” she has experienced for the past few weeks. She says that she has felt really sad, nearly every day and wants to know if she is clinically depressed. Which of the following findings are required to confirm this diagnosis?

a) Fatigue
b) Insomnia
c) Diminished pleasure in almost all activities
d) Excessive guilt or feelings of worthlessness

A

Ans: C

Diminished pleasure in almost all activities; anhedonia

How well did you know this?
1
Not at all
2
3
4
5
Perfectly
95
Q

A 25-year-old male who just finished a four-day camping trip with his parents complains of having been abnormally hungry the past few months. The 6’1” man is especially concerned because his weight has dropped to 140 pounds as well. Lab results reveal two significant findings: hyperglycemia and ketonuria. What is the most likely diagnosis?

a) Hyperthyroidism
b) Hypothyroidism
c) DM type 2
d) DM type 1

A

Ans: D

Diabetes Mellitus type 1

How well did you know this?
1
Not at all
2
3
4
5
Perfectly
96
Q

Allita just gave birth to her first child born in the US. She and her family, which includes six other children, recently immigrated from Mexico. You compliment her baby for doing well and being adorable. The next day, your supervisor gets an angry call from Allita’s husband regarding your behavior. Which of the following is the most likely complaint?

a) In some Mexican cultures, you must always touch the baby when complimenting him or her
b) You did not compliment the other children
c) In some Mexican cultures, only the mother can compliment the child
d) In some Mexican cultures, you should not compliment how a child is doing because that could “jinx” the child.

A

Ans: A

In some Mexican cultures, you must always touch the baby when complimenting him or her.

How well did you know this?
1
Not at all
2
3
4
5
Perfectly
97
Q

Caitlin Jenner, a 46-year-old woman comes to clinic with worries pertaining to a lump she found in her breast. During examination, it is clear she has a mass, although the type is unclear. What is the best diagnostic test to use at this time to distinguish a liquid-filled cyst from a solid mass in the breast?

a) Ductoscopy
b) Ultrasound
c) Mammography
d) Biopsy

A

Ans: B

Ultrasound

How well did you know this?
1
Not at all
2
3
4
5
Perfectly
98
Q

After a car accident, Jo Ann, a 35-yr-old female needs surgery. You are called to coordinate her care. She has refused a blood transfusion because she is a Jehovah’s Witness. However, her husband asks that you give her blood if needed. Of the following, which is the correct choice of action?

a) Follow Jo Ann’s husband’s orders and give the blood transfusion
b) Ensure proper documentation and follow Jo Ann’s refusal of a transfusion
c) Since it is the only method of saving her life, follow the Hippocratic Oath to “do no harm” and give Jo Ann a blood transfusion
d) Wait until Jo Ann loses consciousness and then, administer the transfusion with the husband’s request

A

Ans: B

Ensure proper documentation and follow Jo Ann’s refusal of a transfusion

How well did you know this?
1
Not at all
2
3
4
5
Perfectly
99
Q

Dori, 64, has experienced recurring bouts of shingles. Although you note during today’s appointment that her skin eruptions have disappeared, she complains of pain and burning sensations that remain in the areas of the outbreak. These sensations, she adds, have been severe enough to interfere with her sleep. Which of the following do you prescribe to alleviate the patient’s complaints?

a) Gabapentin
b) Zostavax
c) Acyclovir
d) Famciclovir

A

Ans: A

Gabapentin

How well did you know this?
1
Not at all
2
3
4
5
Perfectly
100
Q

Which of the following cranial nerves does NOT control eye movement?

a) CN VI
b) CN III
c) CN II
d) CN IV

A

Ans: C

Eye movement is controlled by CN III, IV, and VI.

CN II controls pupillary motion and vision

How well did you know this?
1
Not at all
2
3
4
5
Perfectly
101
Q

You are taking the medical history of a 15-year-old Penelope who complains of regularly being tired. You note that she has dry skin, thinning hair, and intolerance to cold. Which of the following should be tested for in order to verify the most likely diagnosis?

a) Ketonemia
b) Elevated TSH and decreased T4
c) Glucosuria
d) Decreased TSH and elevated T3

A

Ans: B

The diagnosis of hypothyroidism is revealed by an elevated TSH and a decreased T4

How well did you know this?
1
Not at all
2
3
4
5
Perfectly
102
Q

A 56-year-old patient comes to clinic complaining of pain and pressure over his face, reaching as far back as his teeth. He also has a fever of 102F which he has been treating with fluids and rest. When asked when these symptoms occurred, the patient said, “about three days ago” which of the following is the most effective way to treat this patient’s complaint?

a) Antibiotics
b) OTC oral decongestants
c) Diazepam for the pain
d) OTC antihistamine

A

Ans: A

This febrile patient needs antibiotics

How well did you know this?
1
Not at all
2
3
4
5
Perfectly
103
Q

A 4 year old girl complains of localized, painful swelling on her left eyelid. Her parents say that an abscess on her eyelid seems to be causing the swelling and redness. Your exam confirms the parent’s suspicions concerning the abscess. Which of the following would be the most likely cause?

a) Conjunctivitis
b) Blepharitis
c) Chalazion
d) Hordeolum

A

Ans: D

Hordeolum is painful, chalazion is painless. The rest are just wrong

How well did you know this?
1
Not at all
2
3
4
5
Perfectly
104
Q

The parents of 9 mo old Kent come to your clinic, hurried and during the middle of the night. The parent states that their child is lethargic and constantly cries out despite all attempts to comfort him. Upon physical exam, the nurse practitioner notes palpable sausage-shaped mass in the right upper quadrant and abdominal distention. When asked about the kid’s stool, the parents stated that his poop was jelly like. Of the following, which is the most likely cause?

a) Pyloric stenosis
b) Appendicitis
c) Malabsorption
d) Intussusception

A

Ans: D

Intussusception

The best way to diagnose this is not US but an X-ray

How well did you know this?
1
Not at all
2
3
4
5
Perfectly
105
Q

Which of the following is dedicated to identifying disparities that prevent people from obtaining health care?

a) Health Insurance Portability and Accountability Act
b) Patient Safety and Quality Improvement Act
c) Healthy People 2020
d) Collaborative Practice

A

Ans: C

Healthy people 2020

How well did you know this?
1
Not at all
2
3
4
5
Perfectly
106
Q

A 36 year old female comes to clinic after getting a positive result on a pregnancy test. She says that she has some dull pain in her lower abdomen, adding that the day before she experienced some spotting in her underwear. During the physical exam, the patient winces when you palpate the left side of her pelvis. Which of the following is the most likely cause of her complaints?

a) Ectopic pregnancy
b) Abruption placenta
c) Pyelonephritis
d) Placenta Previa

A

Ans: A

Ectopic pregnancy

How well did you know this?
1
Not at all
2
3
4
5
Perfectly
107
Q

A 35 year old man presents with complaints of a strange swelling in his throat. He also mentions that he has lost about 12 pounds in the past several months. A Physical Exam reveals that he has swollen lymph nodes and there is involvement in the right axilla and right side of the neck. Based on the suspicion of lymphoma, which stage of disease does this presentation most likely represent?

a) Stage IV
b) Stage II
c) Stage I
d) Stage III

A

Ans: B

Stage II because it has not crossed the diaphragm

How well did you know this?
1
Not at all
2
3
4
5
Perfectly
108
Q

A 32 yr old male marathon runner reports to the urgent care center with severe pain in his RLQ that began shortly after a 15 mile training run. While examining the patient, you find that Freddy clutches his RLQ upon contact and when asked to extend his right leg, he explicitly describes pain. Given this info, which of the following findings would best assist you in differentiating the most likely diagnoses of appendicitis, colon cancer, gastroenteritis, or IBS?

a) His temperature is normal
b) He is experiencing nausea
c) He has experienced vomiting
d) His bowel habits are normal

A

Ans: D

Normal bowel habits rules out most of these conditions

How well did you know this?
1
Not at all
2
3
4
5
Perfectly
109
Q

The parents of a 16 year old are concerned that their kid may be depressed. The parents state that their child has increasingly isolated himself and has an overall sad demeanor. While interviewing the child, which of the following guidelines for effective communication is needed?

a) Telling the patient the exact details for goal resolution
b) Active listening skills and therapeutic response
c) Asking the patient to repeat to you what the school counselor has told them
d) Taking a lead in the conversation and stay on topic

A

Ans: B

DUH! Active listening skills and therapeutic response

How well did you know this?
1
Not at all
2
3
4
5
Perfectly
110
Q

A 27 year old woman presents to the ED with vaginal bleeding. She states that she is 14 weeks pregnant and has not had any prior bleeding or spotting. Of the following signs, which would be the most serious in regard to the patient’s pregnancy?

a) Cramping and pelvic pressure
b) Pink discharge containing mucous plug
c) Frequent or painful urination
d) Constipation

A

Ans: A

Cramping and pelvic pressure

How well did you know this?
1
Not at all
2
3
4
5
Perfectly
111
Q

Sarah brings her 2 year old in for a routine exam. During the exam, she expresses concern about her son’s behavior during the last several months. She says that her son frequently touches his genitals and obsessively pokes at the genitals of his dolls. Additionally, since last month, he has been crying and resisting his mother’s attempts to wipe his bottom after defecation. Sarah works full time and leaves her son at a home run daycare during the week. She is concerned that someone at the daycare might be abusing her son. Which of the following indicates the most appropriate initial course of action the NP must take?

a) Report suspected abuse immediately
b) Call the daycare to investigate
c) Ask general counsel to advise the mother
d) Advise Sarah to ask her son if someone has been touching him inappropriately

A

Ans: A

Report suspected abuse immediately

How well did you know this?
1
Not at all
2
3
4
5
Perfectly
112
Q

Which of the following is the first component of quality assurance?

a) Assessing the effectiveness of an action
b) Delineating the scope of practice
c) Establishing thresholds for evaluations related to the indicators
d) Collecting and organizing data

A

Ans: B

Delineating the scope of practice

How well did you know this?
1
Not at all
2
3
4
5
Perfectly
113
Q

Tanya is 10 years old, 4’8” and weighs 110 pounds. Tanya’s mom is concerned about her weight and eating habits as she has found junk food wrappers hidden around the house. She adds that Tanya is very defiant when it comes to food restrictions. Which of the following is the most appropriate way of approaching the topic of diet with Tanya?

a) Explain that being overweight puts her at an increased risk for developing many types of cancer and heart disease
b) Explain that being overweight can cause type 2 dm, which can lead to major complications including amputations and early death
c) Review the new MyPlate standards with Tanya and help her understand why a balanced diet and maintaining a healthy weight are important
d) Explain that junk food, especially sweets, can lead to tooth decay.

A

Ans: C

Review the new MyPlate standards with Tanya and help her understand why a balanced diet and maintaining a healthy weight are important

How well did you know this?
1
Not at all
2
3
4
5
Perfectly
114
Q

While doing a fundoscopic exam on your patient, you notice several arterioles that cross a venule, producing bulging on either side. Your patient, a 57 year old AA male is slightly overweight and admits to smoking approx. one pack of cigarettes per day. Which of the following is the most likely cause of these findings?

a) Previous trauma
b) Cataract
c) Hypertension
d) Diabetes

A

Ans: C

Hypertension

How well did you know this?
1
Not at all
2
3
4
5
Perfectly
115
Q

Andre, an 89 year old male, enters the hospital with pneumonia. Of the choices below, which details the best way and time to deal with an advanced directive?

a) Advise him as soon as he enters the hospital
b) Advise his family as soon as he enters the hospital
c) Advise him when it seems his symptoms will lead to a coma
d) Advise his attorney when symptoms look like they will lead to coma

A

Ans: A

Advise him as soon as he enters the hospital

How well did you know this?
1
Not at all
2
3
4
5
Perfectly
116
Q

A 31 year old patient comes to clinic complaining of hearing loss but only in the right ear. When examining her, you notice some erosion on her eardrum in the middle ear. Which of the following is the most likely diagnosis and treatment?

a) Serous OM; antibiotics
b) Cholesteatoma; surgery
c) OE; Surgery
d) Cholesteatoma; antibiotics

A

Ans: B

Cholesteatoma indicates a surgical referral

How well did you know this?
1
Not at all
2
3
4
5
Perfectly
117
Q

During a routine PAP smear, 37 year old Angelica requests a birth control prescription and adamantly states that she does not want to conceive at her age. However, during your exam, you note that her blood pressure is at a prehypertensive level. Based on her medical status, which of the following is the safest and most effective birth control option for her?

a) Ortho Evra
b) ParaGard
c) NuvaRing
d) Natural family planning

A

Ans: B

ParaGard

How well did you know this?
1
Not at all
2
3
4
5
Perfectly
118
Q

A 16 year old male, with no abnormalities in his medical records, presents with unusual body proportions and underdeveloped sexual characteristics. A blood test indicates Klinefelter Syndrome. Which of the following is NOT a typical feature of Klinefelter syndrome?

a) Short Stature
b) Hypogonadism
c) Gynecomastia
d) Learning disability

A

Ans: A

Short stature is not characteristic of Klinefelter XXY syndrome.

How well did you know this?
1
Not at all
2
3
4
5
Perfectly
119
Q

Your clinically depressed patient has attempted suicide twice in the last five years. He recently told you that he has been having overwhelming suicidal thoughts over the last week. Which of the following is the best way to proceed?

a) Involuntary commitment
b) Restrain the patient
c) Refer the patient for a psychiatric appointment
d) Contact the local police to have the patient arrested

A

Ans: A

Involuntary commitment since he is a risk to himself.

How well did you know this?
1
Not at all
2
3
4
5
Perfectly
120
Q

A patient presents with a moderate cough and reports experiencing chest tightness as well. The NP notes that the patient has some difficulty breathing. Upon interview, the NP learns that the patient recently moved into a new apartment building that may have cockroaches. Which of the following is the patient most likely experiencing?

a) Acute bronchitis
b) Asthma
c) Pneumonia
d) Chronic Bronchitis

A

Ans: B

Asthma

How well did you know this?
1
Not at all
2
3
4
5
Perfectly
121
Q

A 48 year old female complains of a severe headache with throbbing pain. Her children are entering college, and she admits that she has been drinking alcohol at a higher rate than usual. Furthermore, she states, “I haven’t gotten a good night’s sleep in weeks”. Of the following types of headaches, which is she most likely experiencing?

a) Tension headache
b) Migraine headache
c) Vascular headache
d) Cluster headache

A

Ans: B

Migraine is throbbing

How well did you know this?
1
Not at all
2
3
4
5
Perfectly
122
Q

Jordan, 24, presents with complaints of painful, itchy lesions around her labia majora region. She adds, “I think I’ve had the chills a few times, and i’ve really felt sluggish for the past few days.” Upon exam of the affected area, you note small, painful, fluid-filled blisters. Given her presentation and your assessment, which of the following tests should be ordered to confirm the most likely sexually transmitted disease?

a) Enzyme immunoassay
b) Western blot
c) Gram stain of discharge smear
d) Viral culture

A

Ans: D

viral culture

How well did you know this?
1
Not at all
2
3
4
5
Perfectly
123
Q

During routine physical exam, 35 year old Sarah says that she and her partner are in a committed relationship and are looking for a long-lasting effortless birth control method. She wants to explore her options but mentions that she has an allergy to estrogen-based birth control. Under the circumstances, which of these choices should you recommend?

a) Cervical Cap
b) Ortho Evra
c) Oral contraceptives
d) Implanon

A

Ans: D

Implanon

How well did you know this?
1
Not at all
2
3
4
5
Perfectly
124
Q

A 68 year old male presents to the office complaining of sporadic numbness in his big toe that occasionally affects his entire foot. His history is positive for smoking, hypercholesterolemia, as well as type 2 DM. On physical exam, you note that the skin on his feet and lower legs is shiny and significantly less hairy; however, neither his feet nor legs are remarkably swollen or red. Which of the following is the most likely diagnosis?

a) Chronic venous insufficiency
b) Superficial claudication with ischemia
c) Deep vein thrombosis
d) Peripheral arterial disease

A

Ans: D

Peripheral arterial disease

How well did you know this?
1
Not at all
2
3
4
5
Perfectly
125
Q

For patients with inadequate iron intake, most of what composes a patient’s blood will be lower than normal in lab findings. What aspect of blood however may actually be found to increase?

a) Red cell distribution
b) Mean corpuscular hemoglobin concentration
c) Mean corpuscular volume
d) Serum iron

A

Ans: A

RDW

How well did you know this?
1
Not at all
2
3
4
5
Perfectly
126
Q

A couple brings their 3 year old boy for evaluation because he has been irritable and feverish. Three days prior, they brought him in and were sent home with instructions to give the child acetaminophen. However, his temperature has remained at or slightly above 102F. The parents are also worried about skin rashes found on the child’s trunk and his reddened palms and soles, and point out how red the child’s eyes are “from all the crying”. Your exam reveals a whitish coat on the child’s tongue and very chapped lips. You also see that the skin of the palms and soles is starting to peel. Which of the following is the most appropriate treatment plan for this child’s likely condition?

a) Oral Penicillin; aspirin
b) Admit to hospital for IV gamma globulin and high dose aspirin
c) Acyclovir and acetaminophen
d) Corticosteroids and ibuprofen

A

Ans: B

Admit to the hospital for IV gamma globulin and high dose aspirin. This child has Kawasaki’s disease

How well did you know this?
1
Not at all
2
3
4
5
Perfectly
127
Q

An elderly man presents with swollen lymph nodes and a painless lump in his neck. He also reports that he recently has been feeling generally weak and sometimes has a fever. During the exam, you note tiny, red spots on the skin of his hands. Which of the following is required to confirm the diagnosis that is most likely the cause of his symptoms?

a) Bone marrow aspiration
b) Magnetic resonance imaging
c) Erythrocyte sed rate
d) Peripheral blood smear

A

Ans: A

Bone Marrow Aspiration.

This is the typical presentation of leukemia

How well did you know this?
1
Not at all
2
3
4
5
Perfectly
128
Q

You are examining a new patient. During a review of the chart, you see that he is currently taking inhaled ipatropium bromide. This medication is for what conditions?

a) Asthma
b) TB
c) Pneumonia
d) Emphysema

A

Ans: D Emphysema

Inhaled ipratropium bromide is commonly used in the rx of emphysema to prevent wheezing, difficulty breathing, chest tightness and cough

How well did you know this?
1
Not at all
2
3
4
5
Perfectly
129
Q

A patient is prescribed nitro to alleviate symptoms of chest discomfort that occurs especially during activity such as exercise but also sometimes at rest. The patient’s echo is normal. For which of the following conditions is the patient most likely being treated?

a) Prinzmetal’s angina
b) Stable angina
c) Chest wall pain
d) Myocardial infarction

A

Ans: A Prinzmetal’s angina

Prinzmetal’s angina: symptoms occur during exertion but also occur at rest. Stable angina is excluded because symptom only occur during exertion.

130
Q

Of the following, which is NOT a goal of the “Healthy people 2020” program?

a) Provide healthcare for the uninsured
b) Provide measurable data and goals to guide policy decisions
c) Promote accessibility to health care
d) Help Americans to attain lives free of preventable conditions

A

Ans: A Provide healthcare for the uninsured

Healthy People 2020 does not directly provide healthcare

131
Q

After taking the BP of a 14 year old patient who is in the 90% for height, you suspect that she is Hypertensive. You know that considering her age, her current BP readings most indicate that she has Stage 1 HTN. which of the following BP readings, if present in 3 consecutive readings on 2 separate occasions, would warrant this diagnosis?

a) 136/86
b) 122/78
c) 126/82
d) 142/92

A

Ans: D: 142/92

Pre HTN: 120-139 / 80-89
Stage I: 140-159 / 90-99
Stage II: >160 / >100

132
Q

Reggie, a 30 year old male comes to clinic with a skin infection on his arm. It is as big as a grape. You note that the infection is pus-filled and originated around a hair follicle. Which of the following organisms is the most likely cause of this infection?

a) Enterobacter
b) E. coli
c) S. aureus
d) S. pyogenes

A

Ans: C

Staphylococcus aureus

Likely pathogen is S. aureus. S. Pyogenes is the leading cause of outpatient cellulitis but is not typically associated with a furuncle

133
Q

A fat caucasian, 43, comes to clinic for a check. While reviewing his hx, you see that he has T2DM and 2 previous BP readings of 140/90. Concerned, you check his BP again to get a reading of 142/94. You now consider the patient as hypertensive after the initiation of a thiazide diuretic. Which of the following is the best next step?

a) Beta blocker
b) Calcium channel blocker
c) ACE inhibitor
d) Nitrates

A

Ans: C

ACE Inhibitors

The best choice is an ACE given the history of DM. Betablockers are contraindicated in patients with DM because they mask the normal physiologic body responses to hypoglycemia subjecting the patient to hypoglycemia without warning signs.

134
Q

A 30 year old female comes to clinic with a red and very swollen knee. She is in pain and struggles to walk. She says that she was playing hockey the day before and took a hit to her knee. She says the same thing happened a few months ago and that she already knows it is bursitis. Initially, she had been treated with a steroid injection. At most, how many additional steroid injections should she be allowed this year?

a) 5
b) 2
c) 3
d) 4

A

Ans: B

2 MORE in this year

three total but dumb shit me didn’t see the “how many more” so the answer is 1 + 2 =3

135
Q

Mary, 40, has come to clinic with painful swollen joints. She says that she often feels weak and has lost the desire to eat. You ask her about the intensity of her pain and she says that it is horrible in the morning but gets better as the day goes on. Which of the following single treatments would be least effective in her overall treatment plan in chronic management?

a) Methotrexate
b) Acetaminophen
c) Steroids
d) Gold Salt injections

A

Ans: G

Acetaminophen

RA: Tylenol is the least appropriate med. Methotrexate, steroids, and gold salt injections are all regular parts of RA management

136
Q

To ensure that your research methods are providing intended data, you choose three parameters that you wish to test for and construct three questions as each. After collecting the results, you look for variance within each group of three answers. Which of the following research principles have you appealed to in verifying your research?

a) Validity
b) Internal consistency
c) Reliability
d) Sensitivity

A

Ans: B

Internal consistency measures the reliability of inquiries or instruments to determine if they all are aimed at the right response.
Validity is tested by ensuring that the variable itself is tested accurately and yields the intended information
REliability guages if an instrument will measure results the same way over time.

137
Q

You tell a young woman that you want to prescribe an OC to treat her acne. She says that her previous treatment of topical antibiotics had some unpleasant side effects and asks what some of the negative side effects of using the OC are? Which of the following is a side effect specifically associated with oral contraceptives?

a) Superficial itching
b) Generalized hyperpigmentation
c) Mild swelling
d) Redness to cheeks

A

Ans: B

Generalized hyperpigmentation: melasma

OC may cause brownish blotches or melasma on the skin (hyperpigmentation). Swelling, redness and superficial itching may be side effects of topical acne treatments but are not side effects of the OC

138
Q

Which of the following leukemia types represent 80% of acute leukemia in adults:

a) Chronic myelogenous
b) Chronic lymphocytic
c) Acute non lymphocytic
d) Acute lymphocytic

A

Ans: C
Acute non lymphocytic

Acute Non-lymphocytic or Myelogenous leuk constitute 80% of acute instances in adults. Chronic lymphocytic leuk is the most common non-acute adult leukemia, but the question specifies acute forms.

139
Q

What is the purpose of the reporting system established by The Patient Safety and Quality Improvement act?

a) Provide patients information about healthcare provider error rates
b) Resolve patient safety and health care quality issues
c) List patients with pre-existing conditions to healthcare providers
d) List medical errors of healthcare providers and allocate liability

A

Ans: B
To resolve patient safety and health care quality issues

Purpose of the reporting system is to resolve patient safety and health care quality issues by providing measurable data for research and analysis.

140
Q

Which of the following is the 3rd most common gyn cancer in the US.

a) Vaginal
b) Cervical
c) Ovarian
d) Uterine

A

Ans: B

Cervical cancer is the third.

#1 is Uterine is the most common, 
#2 is Ovarian
#3 is Cervical
141
Q

You are seeing Valerie through her first pregnancy. Her history indicates 2 previous spontaneous abortions, so she is both hopeful and apprehensive. She is particularly curious about Leopold’s Maneuvers. These will help her determine the fetal position of her child?

a) 22 weeks
b) 20 weeks
c) 19 weeks
d) 21 weeks

A

Ans: B
20 weeks

The earliest time a patient can start Leopold’s maneuver is after 20 weeks. The maneuver can be used 21 and 22 weeks into the pregnancy but can be started earlier. 19 weeks is too early. The literature suggests that Leopold’s maneuver is really best if used at 24 weeks, but the answer to this question is that it can be used as early as 20 weeks

142
Q

A patient has used an IUD device discontinuously for the last 10 years. Recently married, she has stopped using birth control and trying to conceive her first child. She asks what risks she may face. What is her risk for an ectopic pregnancy?

a) Approx 50%
b) Approx 15%
c) Approx 9%
d) Approx 30%

A

Ans: C
Approximately 9 %

The risk of ectopic preg in the general pop is 2%. Among those with long term use of IUD, that risk climbs to 9%

Risk to the normal population is 2%
past tubal pregnancy = 15%
hx of prolapsed intervertebral disc = 15%

143
Q

During a regular exam, Alison, a fair-skinned 57 year old asks you to examine a mole on her neck. The area is firm to the touch, evenly pink in tone, irregular in shape, about 1 mm in diameter and somewhat scaly. Which of the following conditions should you suspect?

a) Malignant melanoma
b) Basal cell carcinoma
c) Seborrheic keratosis
d) Squamous cell carcinoma

A

Ans: D Squamous cell carcinoma

This patient’s mole is a papule that matches the appearance of squamous cell carcinoma. Papules are firm, irregular lesions that may be scaly, bleeding and develop in sun-exposed areas.

Seb keratoses are beige brown or black plaques
Malig melanoma are elevated more than 6mm in diameter and vary in color rather than the even pink tone and smaller size of this lesion

144
Q

What clinical stage of syphilis would lymph node swelling appear on a patient?

a) Tertiary
b) Secondary
c) Latent
d) Primary

A

Ans: B Secondary

Lymphadenopathy is a sign of the secondary stage of syphilis. The most visible sign of the primary stage are painless chancres. The latent stage is seropositive but asymptomatic. The 3ary stage presents with serious symptoms of cardiac insufficiency and an aortic aneurysm

145
Q

Which of the following is least likely to pose a risk of genetic problems?

a) A history of premature pregnancies
b) Advanced parental age
c) Family history of birth defects
d) Fetal exposure to intrauterine infections

A

Ans: A
A history of premature pregnancies

Risk: history of advanced parental age, family history of birth defects and fetal exposure to intrauterine infection are all risks

The history of premature pregnancies does not increase the risk of genetic problems.

146
Q

Martin, 22, has been seeing you for migraine treatment. During the visit, you notice his strange stance. You ask about it , and he reluctantly admits to a recent onset of testicular pain. You tell him that it is in his best interest to tell you what is wrong so you can help him. He informs you that he has also experienced thick discharge from his penis along with painful urination. Based on these symptoms, you suspect chlamydia. Which of the following tests would best confirm your suspicion?

a) Enzyme-linked immunosorbent assay
b) Viral culture
c) Enzyme immunoassay
d) Papanicolaou stain

A

Ans: C
Enzyme immunoassay

Enzyme immunoassay is the preferred method used to test for chlamydia however, EIA is also used. This is not such a good question since ELISA and EIA are so close and often interchanged. (http://www.webmd.com/sexual-conditions/chlamydia-tests). Enzyme-Linked ImmunoSorbent Assay (ELISA) is used not in diagnosing chlamydia but rather in diagnosing AIDS or Sickle Cell Anemias.

PAP and Tzanck are used for herpes.
Chlamydia is bacterial, not a virus so a viral panel is not helpful.

147
Q

A patient with OCD comes to the clinic after recently having sex with a prostitute. The patient presents with no findings indicative of any STD but he wants to know which symptoms he should watch for particularly in regards to HIV. Which of the following is traditionally not one of the early signs and symptoms of HIV infection?

a) Weight loss
b) Night sweats
c) Headaches
d) Fever

A

Ans: C

Headaches are not typically symptomatic of early HIV

148
Q

In a test for accommodation, what is the normal reaction of a patient’s pupils when his eyes are following your finger to his nose?

a) The patient’s eyelids will droop
b) The patient’s pupils will slightly dilate
c) The patient’s pupils will constrict
d) One of the patient’s pupils will dilate more than the other

A

Ans: C

The answers will constrict; Think less light entering the eye as focus is taken to the nose

149
Q

You are discussing at-home maintenance of chronic asthma using a metered-dose inhaler. Which of the following is NOT true about using metered dose inhalers?

a) The increased use of a quick-release inhaler should be promptly reported
b) Patients should shake the canister while taking a deep breathe in and out
c) Patients should breathe the medication mist in deeply and hold breath for a count of 10
d) Metered-dose inhalers should not be used with antileukotrienes

A

Ans: D

Metered-dose inhalers CAN be used with antileukotrienes

Not a good question as B: patient should shake the canister… while taking deep breaths can be interpreted as shaking it while inhaling it. nuance but still subj to confusion.

150
Q

You had previously prescribed a 19 year old Alison treatment for her acne. She returns to clinic several weeks later and angrily complains that her skin has not improved. You decide to switch her to a keratolytic treatment. Given these concerns, which of the following pharm agent is best to prescribe?

a) Salicylic acid
b) Tretinoin
c) Benzoyl Peroxide
d) Adapalene

A

Ans: A

Salicylic acid is the key component in Aspirin however, it also is a key component is many derm-acne preparations.

151
Q

A mother brings her 4 year old son to clinic. She is concerned about the blisters on his skin. Upon exam, you find erythema in the affected areas. When you look at him immuniz hx, you see that he had a vaccination four weeks ago and suspect that his rash is an adverse response to the vaccination. Which of the following vacs did he most likely last receive?

a) Inactivated poliovirus vaccine
b) Varicella vaccine
c) Influenza vaccine
d) Pneumococcal vaccine

A

Ans: B

Varicella Vaccine: a reaction of fever and mild rash can occur usually after the first dose but reactions are uncommon (1/20)

152
Q

After being laid off six months ago, your patient Martin, is doing everything he can to save money. He has on occasion, kept food past its expiration date rather than buy more. Now he comes to clinic and presents with signs of a bacterial infection. You decide to prescribe a beta-lactam antibiotic. Which of the following medications should you NOT prescribe?

a) Penicillin
b) Keflex
c) Amoxicillin
d) Bactrim

A

Ans: D

Bactrim is not a beta-lactam

Beta-lactams are penicillins and select cephalosporins including Penn, Keflex, and amoxicillin

153
Q

You begin to notice that your elderly patient’s forgetfulness has been gradually worsening. You suspect dementia and want to find out the cause. Which is the leading cause of dementia?

a) Impactions on the elderly
b) Drug or alcohol abuse
c) Alzheimer’s disease
d) Parkinson’s disease

A

Answer: D Alzheimer’s disease

154
Q

You are doing a well child check on a 7 year old cauc boy. His temp and other VS are normal, but you note only a small height gain since the last visit. The boy remains slightly shorter than his peer group. His father mentions that the boy has been limping and the child tells you that he feels moderate pain in his knee and sometimes in his groin. Further exam reveals limited passive internal rotation and abduction of the hip joint with mild guarding. His father cannot recall nor do you note recent trauma. Which of the following is the most likely dx?

a) Osgood Schlatter disease
b) Hass’ disease
c) Septic arthritis
d) Legg-Calve-Perthes disease

A

Ans: D

Legg calve perthes disease

155
Q

You meet your new patient Ali in the exam room. You see that Ali has drawn a picture of her mother that includes a face, body, legs, arms and hair. Based on this behavior, how old do you think Ali is?

a) 3 years
b) 2 years
c) 4 years
d) 5 years

A

Ans: C

4 years old

156
Q

Michele, 33, visits clinic for an eval of her Sickle Cell Anemia prognosis. Using your knowledge of the anemia and its diagnosis, what is the most reasonable test that you would perform to determine Michelle’s prognosis?

a) Peripheral smear
b) Electrophoresis
c) Schilling test
d) Measure Hemoglobin levels

A

Ans: B

Electrophoresis

Schilling Test is a 4-part test that is used to determine whether the body absorbs vitamin B12 adequately.

Peripheral smear and Hgb levels are too non-specific

157
Q

You are treating Ali, 22, who admits to you that she has never had sex. You know that the administration of a PAP test should start three years after a patient has become sexually active; however, if a female has not yet had sex, like Ali, then the administration of a PAP test should begin/should have begun:

a) A year ago
b) Three years from now
c) When she was 13
d) When she was 16

A

Ans: A

one year ago. PAP is recommended at age 21

158
Q

Which of the following is correct regarding the eruption of permanent teeth?

a) In 25% of people, wisdom teeth will never erupt
b) Generally, all perm teeth except the third molar erupt by age 9
c) Generally all perm teeth except the third molar erupt by age 12.5
d) In 60% of people, wisdom teeth never erupt

A

Ans C

Generally, all perm teeth except the third molar erupt by age 12.5

The third molars are the wisdom teeth

159
Q

Eclampsia is characterized by the findings of pregnancy-induced hypertension and preeclampsia combined with a seizure. There may also be prodromal symptoms such as severe headache plus either epigastric or right upper quad pain. Which of the following would NOT be employed in testing for eclampsia?

a) A 24-hour urine for protein
b) Urine human chorionic gonadotropin
c) Fetal surveillance
d) Complete blood count

A

Ans: B

This woman is already pregnant. NO need to reconfirm

160
Q

A portly Caucasian patient who is 43 comes to clinic for a checkup. While reviewing his history, you learn that he has diabetes and two previous BP reading of 140/90 on 2 different occasions. Concerned, you check his BP again and get 142/94. You now consider the patient as hypertensive. After the initiation of a thiazide diuretic, which of the following meds would be most appropriate for this patient?

a) Beta blocker
b) Ca channel blocker
c) Angiotensin-converting enzyme inhibitor
d) Nitrates

A

Ans: C

ACE inhibitor is the best choice since Beta-blockers are contra in patients with DM since they mask the physiologic signs of hypoglycemia.

161
Q

A mom brings her 1 yr old kid back to clinic following a previous visit for an upper resp infection. Today, the mom tells you that the kid is experiencing a decreased appetite, rapid breathing, along with “fits” of wheezing. Which of the following diagnoses is most likely?

a) Croup
b) Asthma
c) Pneumonia
d) Bronchiolitis

A

Answer: D

Bronchiolitis

Croup is viral
The kid would not have pneumonia unless he had treatment failure for his URI
Asthma is just wrong.

162
Q

A new patient tells you that he has been under the care of an endocrinologist for years. In reviewing his current meds, you notice the drug Fludrocortisone. For which of the following conditions is this patient being treated?

a) Hyperthyroidism
b) Cushing’s syndrome
c) Addison disease
d) Hypothyroidism

A

Ans: A

Addison’s disease

Cushing’s is too much cortisol so clearly we should not add more. Addison’s is inadequate levels of steroid so this drug gives the patient more.

163
Q

After years of not seeking medical treatment, Alan, 69, visits you complaining of lower resp. problems. You suspect that he is asthmatic and that the condition was simply never diagnosed. However, you would like to rule out the possibility of COPD. Which of the following symptoms would be unique to asthma and not to COPD?

a) Tachypnea
b) Pulsus paradoxus > 12mmHg
c) Cough
d) Hyperresonance

A

Answer: B

Pulsus paradoxus > 12 mmHg is a classic sign in asthma

164
Q

You saw Alex and Maya through the birth of their son David. Now, three months later, they say they are ready to resume sexual intercourse. They do not want to conceive but they are against using contraceptive measures, as it conflicts with their religious views. Of the choices, which provides the longest period in which a couple could engage in intercourse with minimized risk of conception?

a) Cervical mucus test
b) Calendar method
c) Basal body temp graph
d) Lactational amenorrhea method

A

Ans: D

Lactational amenorrhea method with no solid food supplementation

165
Q

Which of the following is not a part of the usual management options for patients with an acute presentation of cholecystitis?

a) Intravenous crystalloids and broad spectrum antibiotics
b) Daily nonsteroidal anti inflammatory drugs
c) Gastrointestinal surgical consult
d) Nasogastric intubation

A

Ans: B This patient is acute….we need to intervene

166
Q

Tom, 24, visits clinic complaining of a red rash that has spread very quickly over the past day. The rash is red, warm to touch, and has a glossy appearance. You ask Tom how he feels and he says, “my muscles have been aching since yesterday; in fact, I’ve an altogether ill feeling” you suspect cellulitis, which you know may be caused by one of a few organisms. Which of the following orgs is the leading cause of cellulitis, keeping in mind that Tom is an outpatient?

a) Strep. pyogenes
b) E. Coli
c) Staph aureus
d) Enterobacter

A

Ans: A

Strep pyogenes is the prime organism for community born cellulitis.

167
Q

A new patient comes to clinic seeking assistance in his recovery from a myocardial infarction. In his life, he has dealt with legal ordeals with his family. Even for the treatment he receives from you, he has a written statement of his intent regarding medical treatment. Of the following choices, which is the best description of this document?

a) Living will
b) Healthcare directive
c) Patient self-determination statement
d) Advanced directive

A

Ans: D

This is an advanced directive

An Advanced Directive is a written statement of intent regarding specifics of medical treatment.

A healthcare directive is a type of Adv. Directive that may include a Living Will and/or a Durable Power of Attorney.

168
Q

Alison, 15, has gone from having many close friends at school to being virtually friendless. She does not talk to anyone in the family about it. Her school performance, once stellar, has also declined as she faces difficulty organizing her homework assignments. Upon interview, the NP learns that Alison is aware of the changes but does not believe it to be a problem. She seems only concerned with a boy at school, for whom she has strong feelings. Of these behaviors, which one is NOT considered a developmental warning sign?

a) Poor adjustment to school
b) Difficulty accepting failure
c) Apparent personality change
d) Withdrawal from friends and family

A

Answer: A

Poor adjustment to school

169
Q

A male infant presents with copious purulent discharge from his right eye, so much so that his eye seems to be glued shut. You learn that his mom has an untreated gonorrhea infection. Which of the following is the most appropriate treatment?

a) Azithromycin administered orally
b) Erythromycin administered orally
c) Ceftriaxone administered parenterally
d) Erythromycin ointment

A

Ans: C

Ceftriaxone administered parenterally is the safe treatment for Gonorrhea. If this were Chlamydia, the first line treatment would be Erythromycin eye gtts.

170
Q

What is the “TNM Classification of Malignant Tumors” meant to assess?

a) Types of Cancer
b) Lifespan assessment
c) Probability of malignancy
d) Stages of cancer

A

Answer: D Stages of Cancer

171
Q

Martha is planning a diet overhaul for her whole family to make sure everyone is eating healthier. This overhaul includes changes for her, her husband, and the two teen children ages 14 and 16. As she looks to reduce her family’s calorie intake, what should she be aware of in regards to her children?

a) Nutritional requirements for adolescents are lower than adults
b) She should cut out all protein from the diet
c) Nutritional requirements for adolescents are higher than for adults
d) Nutritional requirements for adolescents are the same as for adults

A

Ans: C

Nutritional requirements for adolescents are higher than for adults

172
Q

A mom brings her 18 month old son in with lethargy and a decreased appetite. After taking a history, you realize that the child has been exposed to lead. Labs reveal the hemoglobin level is 11.2 and the lead level is 46 ng/dL. Which of the following indicates the most appropriate management option for this patient?

a) Order a blood transfusion
b) Notify child protective services
c) Re-evaluate the child in one month
d) Refer to a hematologist for chelation therapy

A

Ans: D

the limit that indicates a need for a heme referral and chelation therapy is 45 ng/dL

173
Q

Which of the following statements is true about an infant patient with a fever?

a) A temp of 101.8 F can be meaningless in the infant
b) Temps over 101.8 F may result in febrile seizures
c) Fever is common in the first few months of life
d) A Fever in an infant is defined as a rectal temp of 100.4F

A

Ans: D

A fever n an infant is defined as rectal temp of 100.4 F

According to the literature, febrile fevers occur at 102 but they can happen at lower temps so I suppose B is also accurate

174
Q

Jackie has a two-day history of an eye infection. Her signs and symptoms include erythema on the right eyelid, localized pain, and tearing. A physical exam reveals a tender mass on the infected eyelid. Which of the following is the most likely diagnosis of Jackie’s condition?

a) Chalazion
b) Hordeolum
c) Bacterial conjunctivitis
d) Blepharitis

A

Ans: B

Hordeolum

175
Q

Mandy comes to clinic with her 1.5 year old toddler Tamara. A former dancer, Mandy is distraught that her daughter may not be able to dance herself unless corrective measures are taken. Tamara is otherwise perfectly healthy. What should the NP advise the mother concerning her child’s bowlegs?

a) Braces for the legs
b) Limited walking
c) Wait and continue to observe
d) Exercise and physical therapy

A

Ans: C

Wait and continue to observe. Genu Varum is not a concern until a child achieves the age of at least 2 years.

176
Q

Which of the following is the most accurate description of the Dubowitz tool?

a) The Dubowitz tool is a gestational assessment tool that evaluates an infant’s reflexes, including Moro, tonic neck, grasping, rooting, and sucking.
b) The Dubowitz tool is used to estimate gestational age by examining the physical characteristics and neuromuscular development of a newborn.
c) The Dubowitz tool is used to assess the physical condition of a newborn at birth.
d) The criteria evaluated with the Dubowitz tool are influenced by labor and birth, therefore, a second exam may be needed to pick up any changes in a newborn.

A

Ans: B

The Dubowitz tool is used to estimate gestational age by examining the physical characteristics and neuromuscular development of a newborn.

177
Q

A disheveled 32 year old Alison comes to clinic. She presents with bruises and a black eye that she said resulted from “running into a door”. However, you suspect a case of possible domestic violence. Which of the following is initially necessary in managing this patient’s care?

a) Immediately report abuse to the police
b) Check the patient’s vitals
c) Refer the patient to a crisis intervention counselor
d) Prescribe selective serotonin reuptake inhibitors

A

Ans: B

Check the patient’s vitals.
Although abuse is unacceptable, this woman is an adult and has the right to manage her own life. The NP should however address the issue openly and offer any support that might be appropriate.

178
Q

A 34 year old male is complaining of pain in his wrist. He says that he is a tailor and that he sews for nearly eight hours a day, and the pain is especially bad by the time he gets home at night. The NP taps a specific nerve on the flexor surface of the patient’s wrist to determine if a tingling sensation radiates from the wrist to the hand. Which nerve did the NP tap?

a) Median
b) Radial
c) Ulnar
d) Medial

A

Ans: A

Carpal tunnel is a disturbance in the function of the Median Nerve

179
Q

Jennifer, 18, comes to clinic for the first time. She mentions that she has now missed her last 2 menstrual cycles. She also tells you that though she is sexually active, she and her boyfriend have always been responsible when it comes to safe sex. She does not want her parents to find out if there is a problem. Of the following, which is the best approach to this situation?

a) Ask the patient to come back after her next anticipated cycle
b) Refer the patient
c) Administer a pregnancy test
d) Tell the patient’s parents

A

Ans: C

Administer a pregnancy test

180
Q

Javier brings his 4 year old infant to a NP because the child has been crying excessively. The NP begins a physical exam of the child measuring his weight and height. What sort of data is the NP obtaining?

a) Metaphysical data
b) Operational data
c) Objective data
d) Subjective data

A

Ans: C

Height and weight are examples of Objective Data

181
Q

George, a 56 year old man, is brought to the ED soon after he had a seizure. His wife tells you that his muscles were repeatedly contracting, after which he collapsed. George describes his collapse as “blacking out”. His wife relates that after his blackout, Geo had more contractions and lost all control of his bowels. The seizure lasted three minutes. Which of the following seizures did the patient most likely experience?

a) Complex partial seizure
b) Absence seizure
c) Status epilepticus seizure
d) Tonic clonic seizure

A

Ans: D

Tonic Clonic Seizure

182
Q

You have been referred a patient who has depression. He presents with symptoms of mild depression, including anxiety and concentration problems; however, he seems to have been steadily improving and is responding to his medication. He does complain that he has been experiencing dry mouth and constipation. Which of the following types of anti depressant medications could be responsible for the patient’s complaints?

a) Serotonin-norepinephrine reuptake inhibitors
b) Monoamine oxidase inhibitors
c) Tricyclic antidepressants
d) Selective serotonin reuptake inhibitors

A

Ans: C

Tricyclic antidepressants

183
Q

A 54 year old woman complains of pain when she breathes and coughs. You palpate her chest and she winces specifically when you touch the region where, in several places, her ribs connect to her sternum. What is the most likely diagnosis?

a) Plantar fasciitis
b) Polymyalgia rheumatic
c) Costochondritis
d) Morton’s Neuroma

A

Ans: C

Costochondritis

UGH I had this when I started yoga! it was horrible

184
Q

Carla, 27, just left a difficult marriage and is back on the dating scene. Mindful of sexually transmitted diseases, she inquires about the vaccines for Hep B, Recombivax HB and Engerix-B. She asks about the schedule of the vaccinations. After the vaccinations are first administered, when should she receive her next two doses?

a) 6 months then 12 months
b) 1 month then 2 months
c) 1 month then 6 months
d) 1 month then 12 months

A

Ans: C

Hep B is given in 3 doses….one now then again in 1 month then the final dose in 6 months.

185
Q

Erin, 23, arrives in clinic five weeks late with her menstrual cycle. She has given birth twice and does not believe she is pregnant now since she is not experiencing any of the previous symptoms such as severe morning sickness. Which of the following is the best next step to diagnose the cause of her condition?

a) Order a serum human chorionic gonadotropin blood test
b) Order an abdominal computed tomography scan
c) Order a pelvic ultrasound
d) Order a serum chemistry panel

A

Ans: A

Order a Serum HCG test

186
Q

A patient comes to clinic complaining of “the worst flu” he has ever experienced. The patient presents with extreme weakness, muscle fatigue, cramps, and cold intolerance, but also constipation and weight gain. You also notice the patient’s dry skin, brittle nails, and puffy eyes. “must be some kind of allergy, or this dry weather here in L.A.” the patient adds. After performing some lab tests, the patient shows elevated levels of thyroid stimulating hormone, T4 is low, and resin T3 uptake is decreased. What is the patient’s diagnosis?

a) Subacute thyroiditis
b) Grave’s Disease
c) Hyperthyroidism
d) Hypothyroidism

A

Ans: D

Hypothyroidism: High TSH, low T4

Subacute thyroiditis is an acute inflammatory disease of the thyroid probably caused by a virus. Symptoms include fever and thyroid tenderness.

187
Q

Sarah, 30, comes to clinic with a red and very swollen knee. She is clearly in pain and struggles to walk. She states that she was playing ice hockey the day before and received a very hard slap-shot to her knee. She tells you the same thing happened only a few months ago and that she already knows it is bursitis. Initially, she had been treated with a steroid injection. At most, how many additional steroid injections should she be allowed this year?

a) 5
b) 2
c) 3
d) 4

A

Ans: B

she got 1 “ a few months ago” and the annual limit is 3 so she can only get 2 more…..1+2=3

the answer is B….2 more

be careful to answer the question asked!!

188
Q

Ty, a 17 year old amateur boxer, is rushed to the ED with a 40 hour history of abdominal pain. Despite the pain, he continued to train for an upcoming boxing match. Last night however, the pain became so severe that it forced him to stop. On exam, you note that there is right lower quadrant guarding with rebound tenderness, pain with his right thigh extension, and pain with rotation of his flexed right thigh. The patient says he has been experiencing nausea. Considering the length of time since the signs began, which of the following should the NP be most concerned about?

a) High fever
b) Perforation
c) Peritonitis
d) Appendiceal abscess

A

Ans: B

Perforation

189
Q

Which of the following contraceptive methods, if prescribed to a female patient, should you recommend that she does not massage the injection site after administration?

a) Depo-Provera
b) Intrauterine device
c) Implanon
d) Ortho-Evra

A

Ans: A

Depo-Provera
Also, Depo should be shaken well to create a homogeneous solution, should be at room temperature, and should be injected into the abdomen or thigh at a 45 degree angle to assure that the medication is deposited in fatty tissue

190
Q

Mary, a 40 year old female, comes to clinic complaining of painful swollen joints. She says that she often feels weak and has lost the desire to eat. You ask her about the intensity of her pain and she states that it is “dreadful in the morning, but get better as the day goes on”. Which of the following single treatments would be LEAST effective in her overall treatment plan for chronic management?

a) Methotrexate
b) Acetaminophen
c) Steroids
d) Gold Salt injections

A

Ans: B

Acetaminophen is the least effective for chronic management of RA

191
Q

Lily, 45 complains of some recent findings that she suspects may be caused by a sexually transmitted disease. She has been urinating frequently and painfully. There has been pain and swelling in her labia and most notably, some green-colored discharge. Which of the following is the best treatment to administer to Lily?

a) Ceftriaxone
b) Cervarix
c) Valacyclovir
d) Azithromycin

A

Ans: A

Lily most probably has Gonorrhea. The treatment for GC is Ceftriaxone.

Cervarix is a vaccine given at age 9-11 for prevention of HPV

Valcyclovir is given for Herpes

Azithromycin is given for the treatment of Chlamydia

192
Q

The parents of 7 year old Jessica state that their daughter has been showing increasingly erratic behavior over the last few weeks. They are concerned about her constant hand washing and an increasingly defiant attitude towards school and other obligations such as going to bed on time. Upon examination of Jessica, you notice that she is constantly blinking and flaring her nostrils. Which of the following is the most probable cause of Jessica’s change in behavior?

a) Sleep disorder
b) Depression
c) Anxiety disorder
d) Tic disorder

A

Ans: D

Tic disorder

193
Q

A 23 year old male comes to clinic complaining of nausea and vomiting, diarrhea and abdominal cramping. Upon reviewing his activity over the past 24 hours, you learn that he consumed supposedly spoiled sour cream. A physical exam indicates his blood pressure is 70/40 mmHg. Which of the following is the most likely cause of his symptoms?

a) Gastroesophageal reflux disease
b) Gastroenteritis
c) Irritable bowel Syndrome
d) Appendicitis

A

Ans: B

Gastroenteritis

194
Q

To ensure that your research methods are providing the intended data, you choose three parameters that you wish to test for and construct three questions for each. After collecting the results, you look for variance within each group of three answers. Which of the following research principles have you appealed to in verifying your research?

a) Validity
b) Internal consistency
c) Reliability
d) Sensitivity

A

Ans: B

Internal consistency

195
Q

You tell a young woman that you want to prescribe oral contraceptives to treat her acne. She says that her previous treatment of topical antibiotics had some extremely unpleasant side-effects and asks what some of the negative side effects of this therapy are. Which of the following is a side effect specifically associated with oral contraceptives?

a) Superficial itching
b) Generalized hyperpigmentation
c) Mild swelling
d) Redness to cheeks

A

Ans: B

Generalized hyperpigmentation

be careful….the OC is systemic and the other answers are local or topical complications….read the question and answers carefully

196
Q

A regular patient of yours comes in for a check. In conversation, he mentions that he just moved to a new apartment that is infested with cockroaches. Which of the following conditions would be most triggered by this environmental factor?

a) Pneumonia
b) Asthma
c) Acute bronchitis
d) Emphysema

A

Ans: B

Asthma

197
Q

You have just diagnosed a 67 year old patient with pneumonia. The patient also has two other conditions: Diabetes Mellitus Type 2, and metabolic syndrome. According to the latest guidelines of the Infectious Disease Society of America and the American Thoracic Society, which treatment would be best for this patient?

a) Doxycycline
b) Azithromycin
c) Clarithromycin
d) Moxifloxacin

A

Ans: D

This patient needs a Fluoroquinolone such as Moxi

Macrolides are not a good choice because of his age and the chance of QT prolongation

Clarithromycin is a huge CP450 3A4 inhibitor

198
Q

You see 12 year old Maggie in clinic who is unable to sleep. She tosses and turns at night and says that she gets very hot. Maggie states that she cannot focus on anything for a long time which was not the case before. Her mom adds that her academic performance has decreased because of her recent inability to maintain focus. The mom also states that Maggie has been more irritable and excitable than usual. You notice a slight tremor in Maggie’s hands when she straightens out her arms. Of the following, which is Maggie’s most likely diagnosis?

a) Type 2 Diabetes
b) Hypothyroidism
c) Type 1 diabetes
d) Hyperthyroidism

A

Ans: D

Hyperthyroidism

199
Q

Jake, 15, comes to clinic about what he hopes is just moderate acne. After examining him, the NP diagnoses Jake with severe pustular acne. Which of the following is the best treatment for the patient’s condition?

a) Salicylic acid preparations
b) Referral to a dermatologist
c) Benzoyl peroxide
d) Retinoic acid cream

A

Ans: B

Jake, because of his pustular lesions, needs to be referred.

200
Q

Which of the following leukemia types represent 80% of acute leukemia in adults?

a) Chronic myelogenous leukemia
b) Chronic lymphocytic leukemia
c) Acute non lymphocytic leukemia
d) Acute lymphocytic leukemia

A

Ans: C

Acute non-lymphocytic leukemia

Be careful. The question asks for “acute leukemia” two of the answers are “chronic”

201
Q

Which of the following disorders occur more commonly in women at a ratio of 8:1 ?

a) Hyperthyroidism
b) Type 1 diabetes mellitus
c) Type 2 diabetes mellitus
d) Hypothyroidism

A

Ans: A

Hyperthyroidism

202
Q

Candy has dealt with very few complications during her pregnancy. Today, she presents with edema, which started in her face but then spread to other parts of her body. She also complains of headaches, particularly pain around her eyes, which she feels is affecting her vision. She also shows signs of jaundice. Which of the following is the patient’s most likely diagnosis?

a) Eclampsia
b) Hemolysis, elevated liver enzymes, and low platelet syndrome
c) Preeclampsia
d) Pregnancy induced hypertension

A

Ans: B

Hemolysis, Elevated Liver enzymes, and Low Platelet syndrome…HELLP syndrome

203
Q

Jo Anne, 65, comes to the office wearing high heeled shoes. She states, “I feel as though I am standing on a pebble”, and she complains of a shooting pain between her toes. What is the most likely diagnosis?

a) Polymyalgia rheumatic
b) Morton’s Neuroma
c) Plantar Fasciitis
d) Costochondritis

A

Ans: B

Morton’s Neuroma

204
Q

Of the following, what causes the most missed days of work in America?

a) Migraines
b) Sinusitis
c) Low back pain
d) Headaches

A

Ans: C

Low Back Pain

205
Q

Claire is in for a check up. She had optic surgery yesterday but realized that her contact lenses were still in her eyes during surgery. She complains that her eye is red and has a clear mucus-like discharge. She also reports a burning sensation and some pain. Her physician prescribed antibiotic eye drops which cleared the symptoms after two days. Which of the following is Claire’s most likely condition?

a) Viral conjunctivitis
b) Corneal ulceration
c) Corneal abrasion
d) Bacterial conjunctivitis

A

Ans: C

Corneal abrasion

206
Q

A 55 year old male comes to clinic complaining of severe pain that is centered in his right eye. He also has nasal congestion in his right nostril. You learn that his symptoms have been occurring several times each week for the past month. Which of the following is the most likely diagnosis?

a) Trigeminal neuralgia
b) Cluster headaches
c) Tension headaches
d) Migraine headaches

A

Ans: B

Cluster headaches

207
Q

A patient comes to clinic and says, “I feel so nervous that I think I am going out of my mind. I also can’t stop shaking, and my heart races all the time”. You learn that she smokes cigarettes, but the patient denies any previous health problems including HTN or diabetes. Of the choices, what would be the most reasonable diagnosis in this situation?

a) Hashimoto’s Thyroiditis
b) Grave’s Disease
c) Hypothalamic deficiency of Thyrotropin-releasing hormone
d) Pituitary deficiency of thyroid stimulating hormone

A

Ans: B

Grave’s Disease: Hyperthyroid disorder

208
Q

Janet is a 30 year old Greek journalist who comes to clinic for a complete physical exam before accepting a long-term post in the Middle East. She says that she is feeling healthy but upon reviewing her blood work, you notice smaller than normal erythrocytes and decreased beta hemoglobin chains. Given the most likely diagnosis, which of the following is NOT an expected finding?

a) Low serum ferritin
b) Decrease in alpha and beta hemoglobin chains
c) Low mean cell hemoglobin concentration
d) Normal total iron-binding capacity

A

Ans: A

Patients with Thalassemia have normal serum ferritin, they make small RBCs of normal color. However, given the smaller size of the RBC, they tend to make many more of them similarly to someone who lives in a high elevation.

209
Q

A friend comes to clinic to chat in private. His relationship with his father is strained and they have not spoken in a number of years. He overheard his mother say that his father returned from his physician and has “abnormal, permanent enlargement of the spaces in this lungs”. Your friend wants to know what condition causes this finding. What should you tell him his father has?

a) Chronic Bronchitis
b) Tuberculosis
c) Pneumonia
d) Emphysema

A

Ans: D

Emphysema: this problem is permanent and treatment should include methods of stopping the progression and minimizing infection

210
Q

Aubrey, 36, is looking for a new form of birth control. You take her history and learn that she had a stroke when she was just 20 years old. Which of the following would greatly increase her risk for a cardioembolic event?

a) Ortho-Evra
b) Copper-releasing intrauterine device
c) Implanon
d) Depo-Provera

A

Ans: A

Ortho-Evra is an Estrogen/Progesterone patch

211
Q

Which of the following is present in over 90% of duodenal ulcers and over 75% of gastric ulcers?

a) Helicobacter pylori
b) Clostridium difficile
c) Staphylococcus aureus
d) Salmonella

A

Ans: A

Helicobacter pylori

212
Q

Which of the following conditions is best evidenced by papilledema?

a) Diabetes
b) Hypertension
c) Vascular hemorrhage and perforation
d) Heart failure

A

Ans: B

Hypertension

213
Q

A 37 year old woman with a history of miscarriages presents to your urgent clinic with vaginal bleeding and cramping. Upon examination, you find that her uterus is enlarged and note the absence of a fetal heart rate. You suspect spontaneous abortion. Which of the following diagnostic tests should be initially performed to confirm this?

a) Human Chorionic gonadotropin levels
b) Cervical cultures
c) PAP smear
d) Hepatitis B surface antigen

A

Ans: A

Human Chorionic gonadotropin levels fall quickly in the presence of a failed pregnancy

214
Q

Of the following patients, which does NOT need to be reported to state or local authorities?

a) Non-rabid dog bite
b) A patient whose serology results are anti-HAV and IgG
c) Accidental gunshot wound
d) A low-risk patient whose tuberculin skin test result is 15 mm

A

Ans: B

This patient is recovered Hep A. IgG anti-HAV antibodies mean that you have had a hepatitis A viral infection. About 8 to 12 weeks after the initial infection with hepatitis A virus, IgG anti-HAV antibodies appear and remain in the blood for lifelong protection (immunity) against HAV.

215
Q

The parents of a 2 month old male bring him to the ED. His parents inform you that he has a history of irritability. He has developed recurrent vomiting and choking while feeding. You ask the parents about the child’s stools, and they respond saying that he has been going less frequently but there are no noticeable changes in the stool itself. Of the following choices, which is the most likely cause?

a) Pyloric stenosis
b) Gastroesophageal reflux disease
c) Meningitis
d) Hirschsprung’s Disease

A

Ans: B

GERD

Hirschsprung’s disease is aganglionic mega colon.
Pyloric stenosis presents with projectile vomiting
Meningitis? NO..

216
Q

Darlene, 38, comes to clinic and says that she has difficulty hearing with her left ear. You conduct the Weber and Rinne tests and the results indicate lateralization to her right ear. Additionally, the test results show that air conduction is better than bone conduction, with a ratio of 2:1 in both ears. The air conduction time of the left ear is 10 seconds and the bone conduction time is 5 seconds. The air conduction and bone conduction time of the right ear is 30 seconds and 15 seconds respectively. What is the most accurate interpretation of these results?

a) The patient may have conductive hearing loss
b) The test results are reflective of normal hearing
c) The patient may have left sensorineural hearing loss
d) The patient may have right sensorineural hearing loss

A

Ans: C

Lateralization of sound to the unaffected side is an indicator of sensorineural hearing deficit. If she has hearing problems on her left and Weber/Rinne lateralize to her right, this is indicative of Sensorineural loss. If this were conductive, the Weber/Rinne would lateralize to the affected side; the left.

217
Q

You and a student NP are reading the chart of a 55 year old female with a history of glaucoma. The chart indicates that the patient was prescribed pilocarpine eye drops for treatment. You ask the student NP to categorize this type of treatment, and he correctly responds with:

a) Beta-adrenergic blockers
b) Alpha-2 adrenergic agonist
c) Carbonic anhydrase inhibitor
d) Miotic agents

A

Ans: D

Miotic agents are cholinergics: These medications reduce eye pressure by increasing the drainage of intraocular fluid through the trabecular meshwork. Cholinergics can be used alone or combined with other glaucoma medications. A combination of medications can help control how much fluid is produced in the eye and increase the amount of fluid that drains out of the eye.

218
Q

Which of the following is the most common cause of bladder obstruction for men over the age of 50 in the United States?

a) Prostate cancer
b) Benign prostatic hypertrophy
c) Epididymitis
d) Acute bacterial prostatitis

A

Ans: B

Benign prostatic hypertrophy

219
Q

8 year old Martin presents with brightly colored cheeks. At first his mother thought Martin was in another fight, but the color had begun to spread to other parts of his body including his upper arms and trunk. Which of the following best describes Martin’s condition?

a) Lyme disease
b) Fifth Disease
c) Rubella
d) Coxsackie virus

A

Answer: B

Fifth Disease also known as Erythema infectiosum or slapped cheek disease
It is caused by Parvo virus and treatment is only for symptoms.

220
Q

Betty visits the clinic with a very red, swollen face. While no purulent drainage is noted, she states that “it looks like I have a very bad sunburn on my cheeks”. When you touch her face, the affected area feels very warm. Given this information and your most likely suspicion, what is the best first-line treatment to provide Betty?

a) Betamethasone
b) Ultraviolet light
c) Penicillin
d) Miconazole

A

Ans: C

Penicillin This is Erysipelas and usually is caused by Streptococcus. The usual approach for this problem is Penicillin however, Clindamycin also works well and will hit both strep and staph

221
Q

Carl, 38, has been diagnosed with hypothyroidism and was prescribed Levothyroxine 3 weeks ago. Today, he presents to clinic with weakness, fatigue, and puffy eyes. His lab findings show an elevated thyroid stimulating hormone level, decreased thyroxine, increased serum cholesterol, and hypoglycemia. Which of the following is the best action to take next concerning Carl’s treatment?

a) Change his medication to Methimazole
b) Continue his current medication regimen without change
c) Increase the oral levothyroxine dosage
d) Change his medication to propylthiouracil

A

Ans: C

He needs his levothyroxine dose increased by 25mcg because he is highly symptomatic for hypothyroidism.

Methimazole is a medication used to decrease the production of thyroid in HYPERthyroidism

222
Q

The NP is reading an interesting article that outlines a new protocol for the management of COPD. She notices that the article was published in a well known peer reviewed journal. Concerning the efficacy of the new protocol, which of the following is the most important to consider before the practitioner adopts this new protocol in her practice?

a) T-test
b) P-value
c) Standard deviation
d) R-Value

A

Ans: B

The probability level is usually at the 95% (.05) or the 99% (.01) level of significance. When a researcher states that a difference between groups is significant at the .05 level, this means that this difference would occur 5 times out of 100 by chance rather than by manipulation of the independent variable. It also means that there is a 95% chance that the difference occurred because of the manipulation of the independent variable. When a difference is significant at the .01 level, it means there is a 99% chance that the difference can be attributed to the independent variable.

223
Q

What is the purpose of the reporting system established by the Patient Safety and Quality Improvement Act?

a) Provide patients information about health care provider error rates
b) Resolve patient safety and health care quality issues
c) List patients with pre-existing conditions to healthcare providers
d) List medical errors of healthcare providers and allocate liability

A

Ans: B

Resolve patient safety and health care quality issues

224
Q

A severely injured patient in a filled-to-capacity emergency room is threatening the safety of the other patients. What is the best initial course of action?

a) Call the police
b) Restrain the patient
c) Call security
d) Eject the patient

A

Ans: C

Call security

225
Q

According to the most recent American Academy of Pediatrics immunization charts, which three vaccines are recommended for all children who are 11-12 years of age?

a) Tdap, 23PS, and Hep A vaccines
b) Tdap, MC4, and Human papillomavirus vaccines
c) Influenza, 23PS, and Hepatitis A vaccines
d) Influenza, 23PS, and MC4 vaccines

A

Ans: B

at 11-12 years, patients should be receiving Tday (not dTap), Meningitis (MC4), and HPV

226
Q

A middle aged patient presents with a fever. According to his chart, he has also lost 25 pounds since the last time you examined him. While in the exam room, he coughs productively and repeatedly at one point, he even coughs up blood. You eventually diagnose him with Tuberculosis. Which of the following regimens would be the correct way to treat this patient?

a) Daily regimen for four months of isoniazid 300 mg, rifampin 600 mg, Pyrazinamide 1.5 gm., followed by four more months of Isoniazid and rifampin daily.
b) Daily regimen for one month of isoniazid 300 mg rifampin 600 mg, pyrazinamide 1.5 gm., and ethambutol 25 mg followed by four more months of isoniazid and rifampin daily
c) Daily regimen for 6 weeks of isoniazid 300, rifampin 600, pyrazinamide 1.5 gm.; followed by four more months of isoniazid and rifampin daily
d) Daily regimen for 2 months of isoniazid 300, rifampin 600 mg., Pyrazinamide 1.5 gm. and ethambutol 15 mg; followed by four more months of isoniazid and rifampin daily.

A

Ans: D Standard treatment for TB is a 6 month course as shown below:

Daily regimen for 2 months of isoniazid 300, rifampin 600 mg., Pyrazinamide 1.5 gm. And ethambutol 15 mg; followed by four more months of isoniazid and rifampin daily.

227
Q

A 3 year old female presents with reports of recent “cold symptoms” according to her mom. On exam, you note that she has a low grade fever, cough and dyspnea. Her lungs are clear on auscultation. Radiographs of the neck show a “steeple shaped narrowing of the trachea”. Which of the following is the most likely diagnosis?

a) Croup
b) Common cold
c) Epiglottitis
d) Group A beta-hemolytic streptococcal infection

A

Ans: A

Croup traditionally shows up on xray as a “steeple shaped narrowing of the trachea”

Epiglottitis shows on Xray as a banana shaped swelling and the child presents with heavy drooling.

228
Q

During a routine exam, your patient collapses. You revive and stabilize him then refer him for an overnight stay in your hospital for observation. Out of the following, which service would qualify as “incident to” billing?

a) The initial office visit
b) The overnight hospital stay
c) Weighing him and taking his temperature
d) The subsequent follow-up exam

A

Ans: C

Weight and VS are considered “incident to” billing and can be charged even though the tasks are delegated to an MA or other office staff.

229
Q

Two adult children come to clinic because they are concerned that their 72 year old father has Alzheimer’s disease. You ask what signs he is exhibiting and are alert to listening for early indicators of the disease. Which of the following is the most common early complaint made by families of persons with Alzheimer’s disease?

a) Forgetfulness
b) Hallucinations
c) Confusion
d) Violent behavior

A

Ans: A

Forgetfulness

though forgetfulness can lead to confusion, the former is the most common initial complaint.

230
Q

Which of the following is least likely to pose a risk of genetic problems?

a) A history of premature pregnancies
b) Advanced parental age
c) Family history of birth defects
d) Fetal exposure to intrauterine infections

A

Ans: A

A history of premature pregnancies does not affect the genetic development of the fetus

231
Q

Colin, 7, comes to clinic with his parents. He has been bed-ridden with bronchitis for the past five days. His father says that Colin was limping yesterday but was able to get out of bed much more easily than the previous four days. Colin tells you that he feels some pain in his left hip. An exam indicates Colin’s temp is normal. While holding the hip in flexion with slight abduction and external rotation, you do not detect any limitation in Colin’s range of motion. When you try the log roll test, you detect involuntary muscle guarding on the left side but no swelling or redness in the hip. Of the following, which is the most likely diagnosis?

a) Toxic synovitis
b) Slipped capital femoral epiphysis
c) Juvenile rheumatoid arthritis
d) Septic arthritis

A

Ans: A

Toxic synovitis often follows a viral illness. Toxic synovitis (also known as transient synovitis) is a common cause of hip pain and limping in children. Doctors don’t know its exact cause, but many kids develop it after having a viral infection (such as a cold or diarrhea). Because of this, some doctors think that toxic synovitis is caused by substances produced by the body’s immune system to fight the infection.

Treatment is with NSAIDs and limitation of activity until symptoms resolve

232
Q

One of the major psychosocial tasks of infancy is:

a) Development of secure attachment
b) Separation individuation
c) Symbiosis
d) Regulation

A

Ans: A

Development of secure attachment

233
Q

When examining Bob, age 10, you note that his upper incisors slightly overlap the lower incisors. The second and lower first molars are absent. Your assessment is:

a) Malocclusion
b) Delayed mandibular dentition
c) Normal dentition
d) Hyperdontia

A

Ans: B

Delayed mandibular dentition as his 1st and 2nd molars should have come in though his 3rd molars should not appear until 9-12 years.

234
Q

Which of the following best describes behavior associated with Piaget’s concrete operations?

a) Learning primarily by trial and error
b) Interpreting events in relationship to self
c) Categorizing information
d) Drawing logical conclusions

A

Ans: C

Categorizing information

235
Q

You would expect a school age child to:

a) Grow 1 inch and gain 5 pounds per year
b) Grow 0.5 inch and gain 2 pounds per year
c) Gain about 6 pounds and grow 2.5 inches per year
d) Height and weight changes per year can not be estimated

A

Ans: C

Gain about 5-6 pounds and grow 2.5 inches per year

236
Q

Johnny comes to clinic. He is 6 years old and has not received any immunizations. Which of the following is not necessary at his age?

a) MMR
b) Dtap
c) HIB
d) Varicella

A

Ans: C

HIB is not given after the age of 5 years

237
Q

You are performing a newborn exam. You notice Brushfield spots in the baby’s eyes. What will you also assess for?

a) Hypertonicity
b) Cherry red spot on the macula
c) Heart murmur
d) Cleft palate

A

Ans: Heart Murmur

Brushfield spots occur in infants with Trisomy 21. Cardiac abnormalities are common in these children. Cherry red macula occurs in children with Tay Sach’s disorder

238
Q

Infants identified as having intrauterine growth retardation are prone to developing hypoglycemia due to:

a) Decreased metabolic rate
b) Low levels of glycogen stores
c) Systemic acidosis
d) Susceptibility to sepsis

A

Ans: B

Low levels of liver glycogen stores

239
Q

A mother of 10 month old Jerry asks about the progression of solid foods into her baby’s diet. From the choices below, you advise adding which food last to her baby’s diet?

a) Egg yolk
b) Egg white
c) Fruits
d) Vegetables

A

Ans: B

the white of the egg is a significant allergen. The yolk is relatively hypoallergenic.

240
Q

During auscultation of a 3-year-old’s heart, you note an irregular heartbeat of 90 beats per minute that slows when the respiratory rate slows and accelerates when the child breathes faster. The rest of the exam is normal. What is an appropriate response to this finding?

a) Record the finding in the child’s chart
b) Order a chest x-ray
c) Order an echocardiogram
d) Refer the child to a cardiologist

A

Ans: A

this finding is typical of Pulmonary stenosis and without symptoms, should just be observed but noted in the chart

241
Q

Which drug is important in the symptom management and the prevention of complications in Kawasaki’s disease?

a) Aspirin
b) Corticosteroids
c) Acetaminophen
d) Penicillin

A

Ans: A

Children with Kawasaki’s disease present with a 5 day high fever. It is one of the few conditions where high dose aspirin for the purpose of reducing clotting. These kids need a cardiac referral for prolonged PR and QT intervals

242
Q

While examining a well-child, you note an innocent heart murmur. The NP knows that which of the following is true about these murmurs?

a) They are best heard in diastole
b) It must radiate to the axilla
c) The intensity is no greater than a II-III/VI
d) There is no variation with changes in position

A

Ans: C

The murmurs must be of grade II-III, they may vary with position change, and are strictly systolic

243
Q

Vomitus that is bilious suggests:

a) Obstruction proximal to the pylorus
b) Obstruction below the ampulla of Vater
c) Pyloric stenosis
d) Peptic ulcer disease

A

Ans: B

Obstruction below the ampulla of Vater

244
Q

7 year old Bobby comes to clinic with inspiratory stridor, drooling, and a temp of 105F. He insists on sitting up during the exam. What is your diagnosis?

a) Foreign body aspiration
b) Reactive airway disease
c) Croup
d) Epiglottitis

A

Ans: D

Children who present with fever, drooling, and an unwillingness to recline should be evaluated for epiglottitis

245
Q

An 8-year-old who presents with a sore throat, tonsils are +4 without exudate and afebrile. The NP should strongly consider:

a) Normal
b) Strep throat
c) Hodgkin disease
d) Pharyngitis

A

Ans: C

An afebrile child with +4 tonsillar enlargement without exudate should raise a flag for Hodgkin Disease

246
Q

During a visit to clinic, Mitch, age 2, presents with large muscular calves and demonstrated difficulty rising from a sitting position. Which lab test is indicated?

a) Serum Calcium
b) Serum magnesium
c) Serum phosphorus
d) Serum creatine kinase

A

Ans: D

Serum creatine kinase is elevated with muscle destruction. the large, muscular, “woody” calves is suggestive of Muscular dystrophy.

247
Q

Tina is 12 years old and presents with a lateral curvature of her spine. Which of the following tests would diagnose scoliosis?

a) Positive Gower sign
b) Negative Gower sign
c) Positive Adams sign
d) Negative Adam Sign

A

Ans: C

Scoliosis is diagnosed by a positive Adam’s Sign. Gower Sign is demonstrated when a child needs assistance from upper extremities to rise to a standing position. This is suggestive of proximal muscle weakness in Muscular dystrophy

248
Q

Which test is not an appropriate test for determining the development dysplasia of the hip in an 18 month old infant?

a) Allis sign
b) Skinfold symmetry
c) Galeazzi sign
d) Ortolani maneuver

A

Ans: ortolani is for hip dislocation ??

249
Q

A CBC ordered for a 10 year old female indicates a low MCHC and a low MCV. Differential diagnosis for this result is:

a) Sickle cell anemia
b) Vitamin B12 deficiency
c) Pernicious anemia
d) Iron deficiency anemia

A

Ans: D

Iron deficiency anemia is microcytic and hypochromic.
B12 and Pernicious anemias are Macrocytic
SSA is hypochromic but with sickled RBCs

250
Q

A 25 year old comes to clinic complaining of decreased mobility and pain of the right shoulder exacerbated by movement. The patient states that he just completed extensive house painting 24 hours prior to the onset of pain. He denies trauma. Passive ROM is intact, no redness or bruising present. What is the next step to be taken in order to make a diagnosis?

a) Palpate structures around the shoulder
b) Order an MRI to evaluate the shoulder
c) Order an X-Ray of the shoulder
d) Request an EMG

A

Ans: A

Least invasive least expensive. We are not finished yet with the assessment.

251
Q

Which of the following signs/symptoms are often associated with headaches due to an intracranial tumor?

a) Pain worse in supine position, focal neuro signs
b) Hyperreflexia; personality changes
c) Acute onset; increasing pain over hours to days
d) Pupillary constriction; stupor

A

Ans: A

Pain is worse in supine due to increases in IC pressure and slow growth of a tumor causes neuro changes types of which are dependent on the location of the tumor.

252
Q

A 39 year old patient was diagnosed with acute bronchitis in the ED and treated with Tylenol, dextromethorphan, and metaproterenol (alupent). The patient’s history reveals a smoking habit of 1 pack per day. He now presents to clinic with a fever of 101.2F and a cough productive of thick yellow-green foul smelling sputum. You would encourage smoking cessation and prescribe:

a) Theophylline
b) A penicillin antibiotic
c) An inhaled corticosteroid
d) A macrolide antibiotic

A

Ans: D

A macrolide antibiotic

253
Q

A 14 year old male with bronchitis is being treated with fluids and expectorants. He returns to clinic with a fever of 103F, right pleuritic chest pain, and green sputum. Which of the following examination results would be expected:

a) Right lower lobe crackles
b) Decreased fremitus
c) Bilateral wheezing
d) Normal percussion

A

Ans: A
Right lower lobe crackles suggestive of pneumonia/consolidation

Fremitus would increase over an area of increased consolidation

254
Q

The follow-up exam of a 12 month old treated 2 weeks ago for bilateral OM reveals a normal left TM and a right TM with visible serous fluid levels and decreased mobility. An appropriate plan includes a:

a) Consultation with an audiologist
b) Prescription for an oral decongestant
c) Follow-up evaluation of the ears in 2 months
d) Referral to an ENT specialist for surgical consult

A

Ans: C

Follow up eval in 2 months. OME is present for 2-3 months after resolution of OM

255
Q

A 32 year old woman reports a 6-month history of intermittent symmetrical swelling of the wrists and daily morning stiffness lasting one hour or more in and around other joints. What is the most likely diagnosis?

a) RA
b) OA
c) Gouty arthritis
d) Reiter’s syndrome

A

Ans: RA

OA is not multi-joint and not symmetrical; also usually presents in older adults and from joint overuse

Gout presents in a distal joint or in the outer ear where cool temps crystalize the uric acid
Reiter’s is “can’t see, Can’t Pee, Can’t Climb a tree” joint pain associated with Chlamydia

256
Q

A 38 year old male comes to clinic with acute abdominal pain radiating to his groin. He is having difficulty walking and is nauseated. He denies recent trauma or previous groin injuries. Exam reveals diffuse swelling of the left testicle and negative cremasteric reflex. The most likely diagnosis is:

a) Testicular cancer
b) Testicular torsion
c) Appendicitis
d) Epididymitis

A

Ans: G Torsion

the only other possible answer is Epididymitis but with that condition, the patient feels relief with the lifting of the scrotum….cremasteric reflex.

257
Q

The NP is suturing a simple laceration on an 11 year old patient. The use of lidocaine with Epi is contraindicated in all of the following areas except the:

a) Scalp
b) Nose
c) Fingers
d) Earlobe

A

Ans: A Scalp

Lido with epi is contraindicated in any area that has limited “one-way” perfusion because the Epi is a strong vasoconstrictor

258
Q

During development of a treatment plan for an 84 year old patient with hypothyroidism, the NP must keep in mind the possibility for the worsening of:

a) Renal insufficiency
b) Osteoporosis
c) Dementia
d) Anemia

A

Ans: B Osteoporosis

there is a high prevalence of bone loss in patients with hypothyroidism treated with exogenous thyroxine.

259
Q

An 18 year old comes to clinic with complaints of maxillary facial pain and yellow nasal discharge for 14 days. What is the appropriate initial pharmacologic intervention?

a) Amoxicillin
b) Diphenhydramine
c) Doxycycline
d) Erythromycin

A

Ans: A Amoxicillin

Amox is first line therapy for the patient with low risk for antibiotic resistance

260
Q

A 10 month old presents with a rash, runny nose, and cough. Exam shows a cluster of tiny white papules with an erythematous areola on the buccal mucosa. What does this suggest?

a) Scarlet Fever
b) Rubella
c) Erythema infectiosum
d) Measles

A

Ans: D Measles

Measles present with buccal Koplic spots (50% of the time but highly sensitive). These kids also present with fever, runny nose, red eyes and spreading rash.

Erythema infectiosum is fifth’s disease and is Parvo
Sixth’s disease is HHV

261
Q

An 85 year old man presents to clinic stating that on his drive over he suffered a sudden loss of vision in his right eye characterized by “a bunch of lights” and a feeling that “a curtain came down”. The most likely diagnosis is:

a) Vitreal hemorrhage
b) Optic nerve lesion
c) Central Retinal nerve occlusion
d) Retinal detachment

A

Ans: D Retinal detachment

Acute onset, “curtain coming down” loss of vision

262
Q

Which of the following criteria differentiates a TIA from a CVA?

a) Focal symptoms
b) Amnesia of incident
c) Slowness of onset
d) Absence of residual symptoms

A

Ans: D

the definition of a TIA is that it lasts no longer than 3 hours and leaves NO residual symptoms

263
Q

Nonviral microorganisms most frequently found to cause acute sinusitis are:

a) Strep pneumoniae and H flu
b) Strep pneumoniae and staph aureus
c) Staph aureus and pseudomonas aeruginosa
d) Strep pneumoniae and pseudomonas aeruginosa

A

Ans: A

Streptococcus pneumoniae and Haemophilus influenzae are the most common non viral pathogens that cause acute sinusitis.

264
Q

A patient is referred with a diagnosis of diabetes, hypertension, and CAD. He is on both insulin and a beta blocker. Assuming the patient will continue the beta blocker, it would be important to educate him on the recognition of hypoglycemia. Which symptom would be most indicative of hypoglycemia in this patient?

a) Edema
b) Tachycardia
c) Palpitations
d) Sweating

A

Answer: D Sweating

Beta-Blockers interestingly block the counter regulatory mechanisms that “warn” of hypoglycemia. The hypo–at least gives the victim some warning that one is coming by causing shakes and pounding pulse. Though this answer is “sweating”, I would support that answer of Tachycardia however the lit supports that HR changes occur after sweating. Hard to say.

265
Q

Which of the following labs are most widely accepted as indicators of the progression of HIV infection?

a) WBC and viral load
b) CD4 and viral load
c) CD4 and Western Blot
d) CD4 and Enzyme-linked immunosorbent assay (ELISA)

A

Ans: B CD4 and Viral Load

ELISA is to detect, Western Blot is confirmatory. Viral load and CD4 count provide important information on virus progression, therapy response, and help determine whether to begin HIV treatment.

266
Q

Trigeminal neuralgia manifests itself primarily with:

a) Electric shock like unilateral facial pain
b) Unilateral facial asymmetry
c) Burning bilateral facial pain
d) Jaw pain extending to the neck

A

Ans: A
Electric shock like unilateral facial pain

unilateral facial asymmetry is Bell’s
Trigeminal neuralgia can burn but not bilaterally. Jaw pain extending to the neck is either CV or a distracter

267
Q

A common rule to follow when prescribing to the elderly is to:

a) Give most medications in the morning to allow slowed body functions to have time to adjust to the drug
b) Start at a lower dose that what is commonly prescribed for adults, and increase the dose slowly
c) Perform Creatinine clearance test for baseline data on all persons over age 70 before starting on new medications.
d) Give medications with a full glass of water to promote absorption and limit harm to the kidneys

A

Ans: B

Because of diminished renal function as we age, always start low and go slow.

268
Q

The NP orders pulmonary rehab for a 75 year old with COPD. Expected outcomes of this program include all of the following except:

a) Enhanced quality of life
b) Increased lung capacity
c) Decreased in-patient hospitalizations
d) Improved exercise capacity

A

Ans: B

in a patient with COPD, the disease cannot be reversed. The damage is there for the duration of the patient’s life. Rehab can serve to improve any complications associated with diminished pulmonary function.

269
Q

Which of the following is the most serious outcome of Barrett’s esophagus?

a) Esophageal adenocarcinoma
b) GERD
c) Peptic stricture
d) Esophageal varices

A

Ans: A

Barrett’s Esophagus can lead to esophageal cancers

270
Q

A 17 year old female is suspected of having polycystic ovarian syndrome. In addition to testosterone, the most appropriate diagnostic test to order would be:

a) BUN, Creatinine, electrolytes, and dehydroeipandrosterone (DHEA)
b) CBC, BUN, Creatinine, and estrone
c) Fasting Blood sugar, CBC, BUN, and sex hormone binding globulin
d) Follicle-stimulating hormone (FSH), lutenizing hormone (LH), prolactin, and TSH.

A

Ans: D

d) Follicle-stimulating hormone (FSH), lutenizing hormone (LH), prolactin, and TSH are lab standards for the diagnosis and confirmation of PCOS. TSH is especially important to rule out thyroid disease as the cause of symptoms.

271
Q

Which of the following is NOT an indication of preeclampsia:

a) Visual disturbances
b) Glucosuria
c) Edema of the face and hands
d) Headaches

A

Ans: B

Patients with preeclampsia present with headache, generalized edema, sudden weight gain, and visual disturbances (sometimes pre-seizure) Also common is trace to +1 proteinuria but glucosuria is not part of this pattern.

272
Q

A patient with T1DM who is on NPH and regular insulin split-dosing presents with complaints of early morning rise in fingerstick blood glucose. A review of an at-home glucose test reveals increased morning levels. After an increase in the evening insulin dose, the problem worsens. This is most likely an example of:

a) Insulin resistance
b) Insulin allergy
c) The Somogyi effect
d) Hyperglycemia-induced hypoglycemia

A

Ans: C The Somogyi effect

The somogyi effect is when there is nocturnal hypoglycemia that stimulates a surge of counter-regulatory hormones which raise the blood sugar. This patient is hypoglycemic at 0300 but rebounds with an elevated BG at 0700. the treatment is to reduce or omit the bedtime dose of insulin.

Alternatively, the Dawn Phenom results from a gradual rise in BG (Slow dawn is rising) tissue desensitization to insulin nocturnally. Treatment for this elevation of 0700 BG is to increase the nighttime dose.

273
Q

To assess ability to think abstractly, you could ask the patient:

a) The meaning of a common proverb
b) What action would be taken if there was a fire in the house
c) To count backward from 100 by 7s
d) To spell a word backwards

A

Ans: A The meaning of a common proverb

274
Q

A 65 year old female presents with shoulder and pelvic girdle pain for the last 6 months. She reports recent unintentional weight loss. On physical exam, there is pain on ROM, with no weakness noted. Lab studies show a low hemoglobin and an elevated SED rate. Which of the following is the most likely diagnosis?

a) Polymyositis
b) OA
c) Polymyalgia rheumatica
d) Fibromyalgia

A

Ans: C Polymyalgia rheumatica

PR is an inflammatory disorder involving pain and stiffness in the shoulder and usually in the hip. treatment is steroids, WBC is usually normal but SED is elevated.

275
Q

A 67 year old patient with T2DM, CHF, and mild CAD is currently taking Digoxin, Hydrochlorothiazide, and atorvastatin. Which of the following is an accurate statement regarding this medication regimen?

a) The glipizide will increase the risk of potassium depletion
b) The digoxin will increase the risk of hypoglycemia
c) The hydrochlorothiazide will predispose the patient to digoxin toxicity
d) The atorvastatin will worsen the diabetes.

A

Ans: ?

When Hydrochlorothiazide and digoxin are taken together, the NP must watch for potassium and Magnesium levels.

Why does this cause Dig toxicity? Look it up

276
Q

You are evaluating a 40 year old patient suspected of having a PE. The patient complains of anxiety and cough. A STAT X-ray is normal. The next test should be?

a) Spirometry
b) MRI
c) Contrast Venogram
d) Helical CT Pulmonary angiography

A

Ans: D Helical CT Pulmonary angiography

277
Q

Which of the following findings is typically a sign of acute appendicitis:

a) Periumbilical ecchymosis
b) Rebound tenderness at McBurney’s Point
c) A Negative Rovsing’s sign
d) Pus and mucus in stool

A

Ans: B Rebound tenderness at McBurney’s point.

Periumbilical pain is where appy pain starts but there should not be bruising.

278
Q

Which of the following GI changes is associated with normal aging?

a) Decreased production of gastric acid
b) Increased salivation
c) Decreased incidence of gallstones
d) Increased esophageal emptying

A

Ans: A Decreased production of gastric acid.

In aging individuals, just about everything decreases but NOT the incidence of gallstones…that is just silly

279
Q

A patient with a diagnosis of diverticulosis presents with localized left lower quad discomfort, a palpable mass, mild leukocytosis, and a temp of 100F. The patient does not appear toxic and can tolerate fluids. An appropriate plan should include:

a) Barium enema and NPO
b) Clear liquids and oral antibiotics
c) Low-fiber diet and referral to a gastroenterologist
d) Hospital admission for IV antibiotics

A

Ans: B
Clear liquids and oral antibiotics

Diverticulosis is the condition of having intestinal “pouches” and may be asymptomatic. If these become inflamed, it may be an infection. Since this patient feels well and should be watched but clear liquids will calm the gut and start to prep the patient for surgery in the event that he perforates.

280
Q

The parent of a 13 year old diagnosed with seizure disorder call to report that the child is exhibiting symptoms of a cold but has no fever. The NP should advise the parent that the development of fever may:

a) Have no effect on the seizure threshold
b) Lower the seizure threshold
c) Make the seizure medication less effective
d) Increase the seizure threshold

A

Ans: B Lower the seizure threshold

Fevers make this patient MORE susceptible to having a seizure: therefore the threshold is LOWERED

281
Q

Which of the following would be appropriate alternative to erythromycin therapy in an 18 year old with Mycoplasma pneumoniae infection?

a) Azithromycin (Zithromax)
b) Cephalexin (Keflex)
c) Amoxicillin
d) Clindamycin (Cleocin)

A

Ans: A Azithromycin

A macrolide is the best choice for an atypical pathogen pneumonia. A floxacin is also appropriate especially for an elder since macrolides are associated with prolonged QT interval, but is not offered as an answer

282
Q

A 66 year old presents with first-time bilateral OME and white patches in the mouth that do not rub off when wiped with a 4 x 4 gauze. The patient should be evaluated for:

a) HIV Infection
b) Myelodysplastic syndrome
c) Congenital lymphoproliferative disease
d) Non-hodgkin’s lymphoma

283
Q

A 17 year old female has never had her menses. She is Tanner stage III of sexual development. Her physical exam is completely normal, and her weight is appropriate for her age and height. What is the most likely diagnosis?

a) Secondary amenorrhea
b) Primary amenorrhea
c) Dysmenorrhea
d) Ovarian dysgenesis

A

Ans: B Primary amenorrhea

I would tell this girl at Tanner III that she should expect her period in the next year. This is primary amenorrhea because she has not yet ever had a period. That is the definition of Primary Amen. HA I made a joke!

284
Q

An 88 year old male presents with concerns about memory loss. He feels good, takes aspirin daily, and has no chronic diseases. He lives alone, drives his own car, and manages his financial affairs. To evaluate his memory, which of the following tests would you choose?

a) Folstein Mini Mental State exam
b) Geriatric depression scale
c) Minnesota multiphasic personality inventory
d) Myers-Briggs Test

A

Ans: A

Folstein Mini Mental State Exam

285
Q

A 50 year old male presents with a chief complaint of malaise. Further questioning reveals that his primary concern is delayed ejacualtion. He is currently taking the following meds: atenolol, paroxetine (Paxil), Loratadine, and HCTZ daily. The most likely cause of the patient’s concern would be:

a) Loratadine
b) HCTZ
c) Atenolol
d) Paroxetine

A

Ans: D

Paroxetine is an SSRI and the side effects of this medication include sexual side effects. This is not such a great question however, because the use of HCTZ has also proven to cause sexual issues.

286
Q

After a 3-week camping trip, an 11 year old is seen for a target lesion with central clearing located in the inguinal area. The patient has had a severe headache, malaise, fatigue, and generalized MSK pain for several days. Pharmacologic management of this condition includes:

a) TMP-SMZ (Bactrim)
b) Azithromycin (Xithromax)
c) Metronidazole (Flagyl)
d) Doxycycline (Doryx)

A

Ans: D

Doxycycline: First line treatment for both Lyme disease and Rocky Mountain Fever is Doxycycline

287
Q

Office spirometry performed with an albuterol nebulizer treatment can confirm a diagnosis of asthma because it indicates:

a) Oxygen saturation
b) If airway obstruction is from pulmonary fibrosis
c) Chronic carbon dioxide retention
d) If a patient has reversible airway obstruction

A

Ans: D

If a patient has reversible airway obstruction. This test differentiates asthma where the symptoms can be reversible from COPD where the changes are permanent

288
Q

The management of choice for Polymyalgia rheumatica includes:

a) Whirlpool
b) Prednisone
c) Analgesics
d) Celebrex

A

Ans: B Prednisone

Polymyalgia rheumatica is an inflammatory disorder involving pain and stiffness in the shoulders and usually the hip. Treatment includes the use of steroids for symptom management

289
Q

Which of the following pharmacotherapeutics would be most important to administer to a patient who has a corneal abrasion?

a) Olopatadine (Patanol)
b) Cromolyn ophthalmic (Crolom)
c) Timolol (Timoptic)
d) Gentamycin ophthalmic (Genoptic)

A

Ans D Gentamycin gtts

The concern with corneal abrasion is infection. Gent gtts is the proper treatment.

Patanol if for allergic conjunctivitis
Cromolyn is a mast cell stabilizer
Timolol is a beta adrenergic receptor blocker for glaucoma

290
Q

During an employment physical exam of a 21 year old female, bruising around the areola on the breasts is noted. An appropriate health history for these findings should include all the the following except:

a) Socioeconomic status
b) Current social relationships
c) History of present or past trauma
d) Mental health status

A

Ans: A

Physical and sexual abuse is not particularly affected by SE status whereas the health history should include investigation of all these other areas.

291
Q

A 15 year old comes to clinic for contraception services 2 weeks after a diagnosis of trichomonal vaginitis, and treatment with 2G of metronidazole (Flagyl). She reports that discharge and itching are gone, but she is urinating frequently with a lot of burning. Patient has not resumed sexual activity and has menstruated since her last visit. Exam reveals mild suprapubic tenderness, no leukorrhea, and a normal wet mount. GC and Chlamy tests are negative. Which diagnostic test should be performed immediately?

a) Urine pregnancy test
b) Microscopic examination of the urine
c) Herpes simplex viral culture
d) Repeat chlamydia test

A

Ans: B Micro exam of the Urine

This patient has signs of a UTI and a culture is indicated. Her other resolved symptoms and recent menstrual period without recent sexual activity rule out the other choices.

292
Q

Which of the following best describes psoriatic lesions in an elderly patient:

a) Localized erythematous vesicles
b) Red, sharply defined plaques with silvery scales
c) Shiny purple smooth lesions
d) Erythematous plaques with central clearing

A

Ans: B Psoriatic lesions are typically defined as red sharply defined plaques with silvery scales that bleed when picked off.

293
Q

A 70 year old male presents with paresthesia of the lower extremities. On exam, he appears pale and shows a decreased vibratory sense. Lab tests reveal elevated indirect bilirubin, Hct = 30%, MCV = 120. The most likely diagnosis is:

a) Pernicious anemia
b) Anemia of chronic disease
c) Folic acid deficiency
d) Iron deficiency anemia

A

Ans: A
Pernicious anemia

Pernicious anemia is macrocytic with neuro symptoms. None of the other anemias have neuro symptoms.
Folic acid deficiency is macrocytic but presents with a big beefy tongue and is usually associated with alcohol abuse. IDA is microcytic and ACD is normocytic/normochromic

294
Q

The best lab test to distinguish iron deficiency anemia from other anemias is:

a) Serum ferritin levels
b) MCHC
c) MCV
d) Transferrin saturation

A

Ans: A
Ferritin levels show the level of iron stores in the body. With IDA, those levels will be low. TIBC will be high because there is a huge closet capacity to store iron and it is empty.

295
Q

A 16 year old female in the first month of taking Ortho-Novum 777 complains of midcycle spotting. She has not missed any doses and uses no other medications. Which of the following is most appropriate:

a) Discontinue use
b) Double dosing x 2 days
c) Change to Ortho Novum 1/35
d) Provide reassurance

A

Ans: D
Provide reassurance

She is only 4 weeks into this new form of OC. Women who use OC may experience breakthrough bleeding and irregular periods for the first three months of use.

296
Q

The most common bacterial agents responsible for pneumonia in older adults residing in the community are:

a) Staphylococcus aureus and H. Flu
b) Strep pneumoniae and H. flu
c) Pseudomonas aeruginosa and S. pneumoniae
d) Moraxella catarrhalis and mycoplasma pneumoniae

A

Ans: B

CAP is most commonly caused by Strep pneumoniae and h. influenza

297
Q

Which of the following would be most appropriate to perform in the initial evaluation of a patient with symptoms of acute prostatitis?

a) UA and culture
b) Scrotal palpation and urine culture
c) Prostate specific antigen (PSA)
d) CBC with differential

A

Ans: A

UA and culture is always done to identify the causative organism and to rule out UTI

298
Q

A 40 year old male presents with a 2 week history of rectal pain and itching. He reports a past history of constipation and finding spots of bright red blood on toilet paper several times a week. Rectal exam reveals a tender, swollen, ovoid mass. The stool guaiac test is negative. Which of the following actions should the NP take?

a) Refer the patient to a gastroenterologist for a malignancy workup
b) Schedule a colonoscopy to rule out colon cancer
c) Repeat the guaiac test three times and obtain a CBC
d) Prescribe a bulk-forming agent and hydrocortisone suppositories

A

Ans: D

bulk forming agents (fiber) will help with the passage of stool, and hydrocortisone suppositories will help reduce the swelling and irritation for this patient with hemorrhoids.

299
Q

An 18 year old college student presents for an athletic physical. When asked about current medications, she mentions that she takes “some herb” she bought at the health food store for migraines and menstrual cramping. Which of the following herbal remedies has been commonly used for these conditions?

a) Ephedra
b) Echinacea
c) Feverfew
d) Ginkgo Biloba

A

Ans: C Feverfew

Ephedra is for appetite control
Echinacea is for immune strength
Ginkgo Biloba is for enhanced memory

300
Q

A 2 month old infant is presented for exam and immunizations. History includes an uncomplicated full term delivery and Hep B virus immunization shortly after birth. Exam is unremarkable except for a diffusely erythematous macular rash in the diaper area, sparing the inguinal folds. No satellite lesions are noted. The infant diaper rash is most likely caused by:

a) Candida albicans
b) Eczema
c) Seborrheic dermatitis
d) Contact dermatitis

A

Ans: D Contact derm

the key to this answer is the sparing of the inguinal folds where the diaper surface is unable to touch the skin. Also, the lack of a rash anywhere else in the body speaks to contact irritation.

301
Q

An 88 year old patient has had a gradual onset of hearing loss in the left ear. Exam shows a large accumulation of cerumen in the external auditory canal. Assuming there is no neural loss, you would expect the Weber test to:

a) Lateralize to the left
b) Lateralize to the right
c) Be inconclusive
d) Not lateralize

A

Ans: A

Lateralize to the affected side with conductive hearing impairment.

302
Q

An 18 month old child presents with a bulging, immobile TM, and a temp of 103F. Assessment also reveals a grade II/VI systolic murmur at the left sternal border. After initiating treatment for OM, the most appropriate interventions is to:

a) Obtain an EKG
b) Obtain and echocardiogram
c) Reevaluate the patient in 10 days
d) Refer the patient to a cardiologist

A

Ans: C
This is the presentation of a child with AOME who also has a stills murmur. Physiologic murmurs are primarily systolic and grade II-III/VI.

303
Q

A 65 year old male presents to clinic complaining of increasing fatigue, dyspnea on exertion, and ankle edema during the day. He has a history of mild HTN for which he saw his physician years ago. The physician advised the patient to decrease his salt intake. On exam, the patient is tachycardic, positive for JVD, and positive for S3 with a systolic murmur. Chest films revealed cardiomegaly with vascular redistribution. The NP diagnosis is:

a) MI
b) Pneumonia
c) CHF
d) Pleural effusion

A

Ans: C Congestive Heart Failure

CHF is associated with an S3 and blood volume changes. An S3 is often found with the fluid volumes found in pregnancy however, it is also heard in the abnormal conditions of hyperthyroidism

304
Q

Research findings have shown that in order to improve the longevity of a patient’s COPD, the treatment of choice is:

a) Oxygen
b) Anticholinergic drugs
c) Systemic steroids
d) exercise

A

Ans: A Oxygen

Oxygen therapy is found to be helpful in patients with COPD and patients are advised to use it >15 hours per day

305
Q

A 47 year old patient presents with complaints of upper abdominal discomfort with nausea and burning after eating. There are no current medications taken. The most likely diagnoses would include:

a) Peptic ulcer disease, colitis
b) Gastritis, Peptic ulcer disease
c) Colitis, small bowel disease
d) Gastritis, Crohn’s disease

A

Ans: B Gastritis/Peptic Ulcer disease

Gastritis and PUD both cause pain when the patient eats due to the increased acid secretion at meals. The symptoms are not consistent with lower bowel disease such as colitis, crohn’s, or small bowel disease.

306
Q

A 20 year old male presents with a 1 month history of a “bump on my testicle”. He reports being sexually active, with frequent episodes of unprotected intercourse. with multiple partners. Which condition should be suspected?

a) Cancer of the testicles
b) Torsion of the testes
c) Acute epididymitis
d) Gonorrhea

A

Ans: A Cancer of the testicles

Torsion and acute epididymitis are painful and present differently.

Gonorrhea presents with a discharge and not a testicular bump

307
Q

To assess spinal function at the S1 level, which deep tendon reflex should be tested?

a) Achilles
b) Cremasteric
c) Anal wink
d) Patellar

A

Answer: A Achilles

Achilles is L5/S1
Cremasteric is for testicular torsion
anal wink is a reflexive contraction associated with S2/S4
Patellar is L3/4

308
Q

A 72 year old female presents with a medical history unremarkable except for the occasional headache. She enjoyed emotional health throughout her marriage and childrearing, although her family history is significant for CVAs and nervous breakdowns. Since her husband died a year ago, she has been waking every morning at about 3 am, is reluctant to go out although she has family member living nearby, and she feels a continual urge to sleep. She cannot discuss her late husband without crying. Differential diagnoses would include:

a) CVA, depression, hyperthyroidism
b) Depression, unresolved grief, hypothyroidism
c) Migraine, anxiety disorder, hypertension
d) Dementia, delirium, pneumonia

A

Ans: B Depression, unresolved grief, and hypothyroidism

309
Q

A patient 28 weeks pregnant reports a single episode of vaginal bleeding. History indicates normal prenatal progress to date, and the patient denies pain, vaginal itching, or discharge. Which of the following is the most appropriate intervention to aid in the diagnosis of this case?

a) Ultrasound
b) Non stress test
c) Nitrazine test
d) Bimanual cervical examination

A

Ans: A Ultrasound

This woman may be presenting with placenta previa. An ultrasound is the least expensive least invasive test

310
Q

Which of the following is the leading cause of Cancer-related deaths in women?

a) Cervical
b) Ovarian
c) Breast
d) Lung

A

Ans: D Lung

The others are in fact killers of women and though all are GYN, in fact, Lung cancer is the #1 killer of women AND of men.

311
Q

A 29 year old male with non-complicated Chlamydia infection may exhibit:

a) Urticarial
b) No remarkable clinical symptoms
c) A green mucoid penile discharge
d) A penile ulcer

A

Ans: B No remarkable clinical symptoms

Urticaria is a rash associated with syphillis if on palmar surfaces
Green mucoid discharge is assoc with GC
a penile ulcer is assoc with syphilis (painless), with HSV, (painful), HIV, Trichomonas,

312
Q

A 2 year old child is diagnosed with radial head subluxation (nursemaid’s elbow). After closed manipulation, the best indication of successful treatment is that:

a) The child quickly begins to use the affected arm
b) A click is felt while the child is extending and rotating the arm
c) Swelling dissipates immediately
d) Pulse and sensation are restored

A

Ans: A The child quickly starts to use the arm

this is usually diagnosed when a child holds his arm close to his body thumb up.

313
Q

Which class of antihypertensive agents has been associated with acute renal failure, and is absolutely contraindicated in patients with bilateral Renal Artery Stenosis?

a) Calcium channel blockers
b) Thiazide diuretics
c) Potassium-sparing diuretics
d) Angiotensin-converting enzyme inhibitors (ACE)

A

Answer: D

ACE inhibitors are contraindicated in patients with RAS and acute renal failure because when GFR is decreased in RAS, angiotensin is the only mechanism by which arterioles in the kidney are kept dilated. ACE inhibitors stop the action of angiotensin and can lead to a shutdown of the kidneys

314
Q

According to the American Diabetic Association, the newest standard for determining the presence of DM based on fasting plasma glucose level is a value equal to or greater than:

a) 126 mg/dL
b) 140 mg/dL
c) 145 mg/dL
d) 160 mg/dL

A

Ans: A 126 mg/dL

315
Q

A 3 year old presents at an inner city clinic with fever, cough, malaise, and loss of appetite. The patient lives with several relatives, including a grandmother who also has a cough. Which of the following diagnostic tests would be most helpful for this child?

a) Mantoux PPD skin test
b) Sputum culture
c) Throat culture
d) Cold agglutinin test

A

Ans: A This child should be tested for TB
OR Answer B: Sputum culture

I got this answered both ways in various tests. TB is confirmed by a culture and an Xray. Seems a sputum is the correct answer with TB in the differential. Tough to choose.

a cold agglutinin disease is an autoimmune disease where the presence of high concentrations of circulating antibodies are directed against RBCs

316
Q

A 1 week old is brought to the clinic because of drainage from the infant’s left eye. The parent states that the infant has had the condition since birth, and the it might be “pink eye” needing antibiotic treatment. The NP should:

a) Ophthalmic gentamicin with a follow up in 1 week
b) Neomycin polymyxin (cortisporin) suspension, with follow up with an ophthalmologist
c) Topical erythromycin, returning for eval in 2 days
d) Ketorolac drops (acular) with follow up in 3 days.

A

Ans: C

Topical erythromycin and reevaluate in 2 days.
This child at 1 week probably has exposure to Chlamydia. First line treatment is topical erythromycin. Alternatives are oral macrolide or Doxy
We are assuming that the child got GC eye gtts prophylaxis at birth

Acular is a steroid for allergic inflammation

317
Q

Assessment findings that would support a diagnosis of mitral regurgitation include:

a) Systolic murmur with split S1
b) Loud, high pitched pansystolic murmur
c) Atrial fibrillation with summation gallop
d) Early diastolic murmur with an S3 wave

A

Ans: B Loud high pitched pansystolic murmur

We should ignore all the loud, high pitched, singing, descriptors and remember Ms ARD and Mr ASS

318
Q

An 88 year old presents with right-sided weakness after being unable to rise unassisted following a fall to the bathroom floor. History includes aphasia and noncompliance with a hypertension medication regimen. What is the most likely diagnosis?

a) Seizure
b) Right sided CVA
c) Left sided CVA
d) TIA

A

Ans: C Left sided CVA

The left side of the brain controls the right side of the body. Right sided weakness suggests left sided brain incident.
TIA leaves no residual physical affect
Seizure is just wrong

319
Q

A 14 year old who fell on her outstretched hand complains of proximal forearm pain. X-ray reveals a positive fat pad sign and the patient is unable to fully extend the elbow. No bony changes are seen on Xray. The most likely diagnosis is:

a) Upper extremity sprain
b) Lateral epicondylitis
c) Radial head fracture
d) Olecranon bursitis

A

Ans: C Fracture of the radial head

The fat pad sign is characteristic of a radial head injury or an occult elbow fracture; even when xray does not show any sign of fracture. The displaced fat pads, a useful sign of an intraarticular fracture of the elbow, is the clear depiction of displaced humeral capsular fat-pads

320
Q

A patient with macular degeneration has difficulty seeing objects:

a) From a distance
b) In the center of the visual field
c) At reading distance
d) In the peripheral fields

A

Ans: B Vision loss occurs in the middle of the visual field

Presbyopia and Glaucoma cause a loss of vision in the peripheral fields. Glaucoma is called the silent thief of vision

321
Q

A 4 year old girl with a history of recurrent urinary tract infections returns for follow up after diagnosis of the most recent infection. During the physical exam, the 4 year old tells the NP that her brother, age 9, lies on top of her and rubs his penis near her pubic area. After completing the exam, the NP must:

a) Discuss the child’s report with the mother
b) Report suspicions to the appropriate child protective services agency
c) Refer the case to a physician
d) Suggest psychological counseling

A

Ans: B

Mandatory reporting rules require that we report suspected physical or sexual abuse to the appropriate agency when suspicions include children or the elderly (dependent individuals)

322
Q

Which of the following wet-mount results confirms a preliminary diagnosis of bacterial vaginosis?

a) Squamous epithelial cells with stippling appearance and indistinct borders, no lactobacillus rods, many white blood cells
b) Squamous epithelial cells with clear cytoplasm and distinct borders, many lactobacillus rods, occasional WBCs
c) Organisms about the size of WBCs with undulating flagellum, occasional lactobacillus rods, many WBCs
d) Hyphae and spores, few lactobacillus rods, occasional WBCs

A

Ans: A Squamous epithelial cells with stippling appearance (Clue cells) and indistinct borders, no lactobacillus rods, many white blood cells.

Stippling is characteristic of Clue cells and diagnostic of BV. With this bacterial infection, WBCs should be expected. The most widely accepted clinical criteria are ‘Amsel’s criteria’. This clinical diagnosis requires that three of the following four criteria be met: first, a vaginal pH of greater than pH 4.5; second, the presence of clue cells in the vaginal fluid; third, a milky, homogeneous vaginal discharge; and finally, the release of an amine (fishy) odor with KOH

The large gram-positive bacilli are Lactobacillus species, which are the predominant normal flora of the female genital tract